Наука


Ответить в тред Ответить в тред

<<
Назад | Вниз | Каталог | Обновить тред | Автообновление
525 45 101

Тупых вопросов тред. Аноним 13/01/20 Пнд 06:07:02 4928831
image.png (137Кб, 399x400)
399x400
Тупых вопросов тред.

Прошлый: >>489321 (OP)
Аноним 13/01/20 Пнд 06:09:29 4928852
>>492880
>Я не говорю про гипотетическую, воображаемую вставленную которую которая сродни бредням религоблядей никто никогда не видел и не увидит.
Сами объекты там - не увидим, но судя по тому, что галактики, звезды и все остальное ведут себя так же, независимо близко они к нам, или нет, мы можем сказать, что эта "гипотетическая, воображаемая вселенная" примерно такая же, как тут, и там тоже. Что делает ее гораздо менее гипотетической или воображаемой. Я тебе привел даже пример, как с помощью гипотетических братьев по разуму - мы можем заглянуть за эти 46.
>Так расширение и не даст передать мне недостижимую инфу. Все уйдет в красный спектр и далее.
Проснись, ты серишь, куда у тебя за 1 млрд. св. лет все уйдет в красное смещение?
Аноним 13/01/20 Пнд 06:10:13 4928863
>>492883 (OP)
Если радиус наблюдаемой Вселенной, в световых годах, больше её возраста,
значит ли это что кусок Вселенной находится за "световым конусом"?
Аноним 13/01/20 Пнд 06:10:20 4928874
Аноним 13/01/20 Пнд 06:10:42 4928885
Аноним 13/01/20 Пнд 06:11:36 4928906
>>492880
>Относительные координаты у этого последнего рассеяния есть

Лол, они есть для каждой точки в пространстве. И от каждой точки в пространстве до поверхности последнего растения сейчас 46 миллиардов световых лет. А 10 миллиардлв лет было гораздо меньше чем 46 миллиардов световых лет, а через 10 миллиардов лет будет гораздо больше чем 46 миллиардов световых лет.
Аноним 13/01/20 Пнд 06:19:59 4928917
>>492885
> Сами объекты там - не увидим, но судя по тому, что галактики, звезды и все остальное ведут себя так же
В квазикристаллах тоже видим только трехмерные объекты, а формулы почему-то 6-мерные. Хрупкая ваша гипотетическая дорожка, ладно бы мы могли еще поверить а так проверить невозможно.
> Проснись, ты серишь
Дошло.
Аноним 13/01/20 Пнд 06:25:55 4928928
>>492891
>Хрупкая ваша гипотетическая дорожка
Пока нет данных, которые опровергают. А если бы там физические законы менялись/ничего не было - мы бы наблюдали иное поведение объектов. Как с Нибиру, даже если бы она "пряталась" за Солнце, поведение (орбиты) других объектов тут же выдало бы нам, что там есть масса.
>а так проверить невозможно
Смотря что проверять. См выше.
Аноним 13/01/20 Пнд 06:31:19 4928939
map1b.gif (166Кб, 811x184)
811x184
Есть 3 можно так сказать границы во Вселенной
1. горзон частиц
2. горизонт событий
3. сфера Хаббла

Пикрелейтед по горизонтальной оси отложено сопутствующее расстояние, по вертикальной оси отложено конформное время.

Горзонт частиц - это расстояние до самого далекого источника, в принципе наблюдаемого в данный момент времени (на всякий случай уточним, что речь идет о расстоянии до объекта в момент приема фотона, а не в момент излучения!). Иногда этот радиус определяют по-другому: расстояние, которое фотон может пройти от t=0 до данного момента (т.е. это расстояние, на которое можно передать информацию за время, равное возрасту Вселенной). Обратите внимание, красное смещение источников на горизонте частиц бесконечно.

На рисунках горизонт частиц проиллюстрирован световым конусом из точки t=0, χ=0 в будущее. Однако, этот конус сам по себе не является горизонтом частиц! В каждый данный момент ti горизонт является сечением этого конуса плоскостью t=ti. Т.е. это трехмерная сфера вокруг нас, которая изменяется с течением времени. Зато нарисованный конус позволяет увидеть, как горизонт частиц изменяется со временем (в частности, как "галактики" входят в него, т.е. становятся видимыми для нас).

Горизонт событий - довольно хитрое понятие (и не во всякой космологической модели он существует). Давайте еще раз посмотрим на пик. Кроме нашего светового конуса (для настоящего момента времени), мы видим световой конус для момента в бесконечном будущем - это и есть горизонт событий. Он делить плоскость (пространство-время) на две части. События внутри конуса (напомним, что точка на этой плоскости - это именно событие в пространстве И времени) делятся на две группы. Те, что находятся внутри конуса или были доступны нам для наблюдения в прошлом, или же будут доступны в будущем. События вне конуса нам принципиально недоступны для наблюдений.

Сфера Хаббла - область расширяющейся Вселенной, окружающей наблюдателя, за пределами которой объекты удаляются от наблюдателя со скоростью большей, чем скорость света.

Ничего не понятно? Тогда го http://www.astronet.ru/db/msg/1194830
Аноним 13/01/20 Пнд 06:35:04 49289410
>>492889
Только чем больше расстояние до границы, тем меньше наблюдаемой материи внутри этих границ. Чтобы передать мне инфу что находится за моим личным ГС надо отослать сигнал на 46млрд + красное смещение.
Аноним 13/01/20 Пнд 06:37:46 49289511
>>492893
> и не во всякой космологической модели он существует
Например?
Аноним 13/01/20 Пнд 06:49:01 49289612
>>492886
Бамп! 46 млрд. световых лет не пролетел бы свет за 13.7 млрд. лет.
Аноним 13/01/20 Пнд 06:55:03 49289713
>>492896
Р А С Ш И Р Е Н И Е
А
С
Ш
И
Р
Е
Н
И
Е
А вообще читай еще раз пост вот этого адеквата: >>492893
>>492894
>чем больше расстояние до границы
>тем меньше наблюдаемой материи внутри этих границ
Шизофазия? Выход есть, прими таблетки.
Аноним 13/01/20 Пнд 06:55:10 49289814
>>492886
>Если радиус наблюдаемой Вселенной, в световых годах, больше её возраста,
>значит ли это что кусок Вселенной находится за "световым конусом"?

За "световым конусом" т.е. горизонтом частиц >>492893
>На рисунках горизонт частиц проиллюстрирован световым конусом из точки t=0, χ=0 в будущее. Однако, этот конус сам по себе не является горизонтом частиц! В каждый данный момент ti горизонт является сечением этого конуса плоскостью t=ti. Т.е. это трехмерная сфера вокруг нас, которая изменяется с течением времени.
Аноним 13/01/20 Пнд 07:03:56 49289915
>>492897
> Шизофазия?
Нет! Расширение же ускоряется и со временем становится все меньше высокочастотных фотонов.
Аноним 13/01/20 Пнд 07:10:21 49290016
>>492899
>> Шизофазия?
>Нет!
Да. У меня хомяки выдает примерно такой же по осмысленности текст, когда верещат.
>Чем больше расстояние до границы
Откуда?
>тем меньше наблюдаемой материи
Как это связано?
> внутри этих границ
Их уже стало много, границ-то.
>Расширение же ускоряется
Первая осмысленная часть текста.
>со временем становится все меньше высокочастотных фотонов
Но дальше снова набор букв.
Аноним 13/01/20 Пнд 07:13:05 49290117
>>492900
> Откуда?
От точки наблюдения.
> Как это связано?
Чем больше проходит времени, тем больше становится пространства, оно же расширяется, т.е. тем меньше становится "плотность".
Аноним 13/01/20 Пнд 07:14:22 49290218
Плотность материи в пространстве.
Аноним 13/01/20 Пнд 07:24:50 49290419
>>492901
Средняя плотность упала, ок. Ты это все к чему?
Аноним 13/01/20 Пнд 07:27:03 49290520
>>492904
Что границы сжимаются.
Аноним 13/01/20 Пнд 07:28:01 49290621
>>492905
Жди, когда отпустит - перестанут.
Аноним 13/01/20 Пнд 07:32:32 49290722
>>492906
В голос. Три часа назад еще декса закинул В моих же суждениях нет ошибки , голактеги так быстро разлетаются друг от друга, что эту границу можно переносить все "ближе".
Аноним 13/01/20 Пнд 09:19:15 49290923
>хлебушки, которые про дифгем и дифуры краем уха слышали из научпоп высеров, обсуждают ОТО
и это в свежем треде, обос/sci/ off to a good start
Аноним 13/01/20 Пнд 11:48:18 49291424
>>492909
А ты умеешь в решение дифференциальных уравнений?
Аноним 13/01/20 Пнд 11:57:36 49291625
>>492914
Любой первокурсник умеет.
Аноним 13/01/20 Пнд 12:11:15 49291826
>>492916
>Любой первокурсник умеет.
Я не про любого, я про тебя спросил. Ты умеешь?
Аноним 13/01/20 Пнд 13:11:23 49292227
>>492918
ушел читать дифуры
Аноним 13/01/20 Пнд 13:20:44 49292328
Аноним 13/01/20 Пнд 14:42:21 49293429
8720ccu-480.jpg (38Кб, 350x264)
350x264
Аноним 13/01/20 Пнд 15:20:21 49293830
>>492918
>Я не про любого, я про тебя спросил. Ты умеешь?
Ну а как же.
Аноним 13/01/20 Пнд 15:32:22 49294031
Аноним 13/01/20 Пнд 23:33:55 49299132
Возможно ли управлять климатом, подвесив большой экран на ГСО, который будет отражать ик-излучение, падающее на жаркие регионы? Или это крайне незначительно повлияет, не позволит направлять/разъёбывать циклоны-антициклоны вообще никак? Или его быстро продырявит всякой пылью?
Аноним 14/01/20 Втр 00:17:39 49299333
Аноним 14/01/20 Втр 00:32:36 49299434
Аноним 14/01/20 Втр 00:48:08 49299535
>>492991
Нет. Ты как себе этот экран представляешь на ГСО? Зонтик диаметром с луну видимо? А ГСО? Впрочем я знаю, один хуй, что неправильно представляешь.
Да и ебошить лучом смерти с орбиты - не самая лучшая стратегия управления климатом.
Аноним 14/01/20 Втр 11:18:51 49302136
Можно ли объяснить эффект Манделы путешествиями во времени?
Аноним 14/01/20 Втр 13:27:11 49303137
>>493021
Времени не существует, путешествовать некуда
Аноним 14/01/20 Втр 13:34:51 49303338
image.png (409Кб, 506x604)
506x604
Аноним 14/01/20 Втр 14:00:31 49303739
220px-Clifford-[...].gif (3886Кб, 220x220)
220x220
>>493033
А что есть время?
Аноним 14/01/20 Втр 14:26:09 49304340
Существует ли в теории возможность запилить небольшой компьютер размером с мобилку, который бы работал как мозг (мыслил, обучался)? Словно чип терминатора?
мимо не знаю даже как работает обычный компьютер и что из себя представляет электроток
Аноним 14/01/20 Втр 14:27:02 49304441
Аноним 14/01/20 Втр 14:39:05 49304542
>>493043
Да. Мозг это всего-лишь нейросеть, а животные обладающие нервной системой не более чем биороботы.
Аноним 14/01/20 Втр 16:00:15 49305243
sci-hub tw у кого-то работает?

Второй день:
504 - Gateway Timeout

Другие адреса редиректят на tw.
Аноним 14/01/20 Втр 16:20:33 49305444
>>493052
Через ТОР тоже не открывается. вонючий капитализм.

Что случилось?
Аноним 14/01/20 Втр 16:43:17 49305845
Аноним 14/01/20 Втр 16:43:36 49305946
>>493058
Эта формация реально способна дать капитализму пососать.
Аноним 14/01/20 Втр 16:44:10 49306047
>>493059
В том числе и глобальному рынку, то есть капиталистоблядкам-глобальщикам.
Аноним 14/01/20 Втр 18:02:13 49306248
Аноним 15/01/20 Срд 03:46:28 49308149
>>493052
>>493054
Через бразильские, индонезийские и иранские прокси сегодня открывал. Походу на территории России заблочили.
Аноним 15/01/20 Срд 05:03:00 49308650
>>493052
У меня пашет через тор. Надо только подождать, когда он как следует заведётся.
Аноним 15/01/20 Срд 09:14:31 49309251
Что вообще по генной инженерии растений сейчас известно? Почему не запилят дико быстрорастущие деревья, чтобы ои весь CO2 из атмосферы выжирали да ещё древисину имели какую-нибудь шикарную-технологичную? Можно ещё им дырочку снизу между корней сделать, чтобы они просто ствол свой скидывали в определённый момент и начинали растить новый.
Аноним 15/01/20 Срд 09:48:27 49309452
>>493092
>деревья, чтобы ои весь CO2 из атмосферы выжирали
А ты в курсе, что растения дышат кислородом и выдыхают углекислый газ?
Аноним 15/01/20 Срд 09:53:20 49309553
>>493094
Растения-двачеры?
Аноним 15/01/20 Срд 09:58:54 49309654
>>493095
Все высшие растения (травы, кустарники, деревья и т.д.) вдыхают кислород и выдыхают углекислый газ.
Аноним 15/01/20 Срд 10:06:16 49309955
>>493096
А если их постоянно освещать.
Аноним 15/01/20 Срд 10:10:52 49310156
>>493099
>А если их постоянно освещать.
Не получится. Земля вращается вокруг свое оси и происходит смена дня и ночи.
Аноним 15/01/20 Срд 10:43:45 49310857
При охлаждении тела энергии затрачивается больше чем при его нагревании?
Аноним 15/01/20 Срд 10:48:35 49310958
>>493101
Люди давно придумали ночное освещение.
Аноним 15/01/20 Срд 10:51:28 49311059
>>493109
Люди придумали как сделать ночное освещение всей поверхности Земли занятой растениями? А сколько для этого, кстати, потребуется энергии, можешь посчитать? Для упрощения возьми всего 1 континент и посчитай сколько нужно энергии для ночного освещения всех лесов на этом континенте.
Аноним 15/01/20 Срд 10:57:50 49311360
>>493110
Недостаточно данных.
Аноним 15/01/20 Срд 11:06:08 49311561
>>493113
ОК, посчитай по одной стране. Площадь лесов Канады 244,6 млн.га. Целевая освещенность 300 люкс.
Аноним 15/01/20 Срд 19:00:04 49317062
>>493094
Ну я и спрашиваю - что по генной инженерии?
Аноним 15/01/20 Срд 23:55:00 49319963
Насколько удобно использование электронвольта, или он считается "прошлым веком"?
Где находится граница между квантовой физикой и физикой макромира?
Насколько теоретически обоснованы виртуальные частицы и почему они появляются?
Как "начинают" изучать квантовую гравитацию - смотрят на природу взаимодействия квантового мира, из общего числа явлений вычитают те, которые уже знают, почему происходят, а потом исследуют те, которые незнамо почему происходят?
Интелллект и истинкт - враги?
Раз человек продукт работы и инстинкта, как низменного, и интеллекта, как возвышенного, то получается, что вся наша жизнь - ни материальна, ни духовна; ни полна счастья, ни утопает в страданьях?
Почему все политические идеологии стремятся к утопии, но лишь разъединяют людей между собой?
Почему сериал "Сверхъестественное" пишется с "ъ", хотя должен писаться без "ъ", так как это исключение из правил (иностранная приставка)?
Есть ли у физики потолок? Что будет, когда мы достигнем его?
Наблюдается ли разделение между физикой макромира и мегамира, как по отношению физики микромира и макромира?
Насколько этична будет принужденная евгеника, когда слабые, гневные и "жалкие" люди будут лишены возможности размножаться? Можно ли жертвовать миллиардами нынешних людей ради будущего всего человечества?
Теории Заговора - вызваны ли они слепой надеждой на величие человеческого разума? Ведь большинство заговорщиков пытается оправдать несовершенство человека сторонними силами, а не принять их.
Можно ли при относительно нормальным условиях, с неплохим разумом, но под давлением эмоционального груза ёбнуться головой?
На чём лучше сконцентрироваться - улучшение уровня жизни беднейших и просвещать их или же продолжать крупные исследования неизведанных разделов наук, обращая меньшее внимание на образование в среднем по стране?
Есть гипотетические предположения, какая материя могла бы получиться из второго или третьего поколения кварков?
На чём основываются предположения о многомерности реальности (4+ измерения, а в Теории Струн, если не ошибаюсь, 11)? Обязательно ли они должны быть связаны со временем?
Как сильно изменится наше понимание о физике, если CPT-чётность будет опровергнута?
Из чего состоит бозон Хиггса?
Аноним 16/01/20 Чтв 00:31:42 49320064
>>493199
Ебать ты душный...
Аноним 16/01/20 Чтв 00:47:25 49320165
>>493199
>Насколько удобно использование электронвольта
В физике высоких энергий, в физике конденсированных состояний пользуются этой единицей, значит она им удона
>Где находится граница между квантовой физикой и физикой макромира?
Верхняя масштабная граница микромира примерно соответствует размерам атомов - 10-8 см.
>Насколько теоретически обоснованы виртуальные частицы и почему они появляются?
В рамках теории вполне обоснованы.
>Раз человек продукт работы и инстинкта, как низменного, и интеллекта, как возвышенного, то получается, что вся наша жизнь - ни материальна, ни духовна; ни полна счастья, ни утопает в страданьях?
Это точно не к науке вопрос.
Аноним 16/01/20 Чтв 01:54:41 49321466
>>493199
>Как "начинают" изучать квантовую гравитацию
На бумаге в математике. Если ты про эксперементы, то гравитация на столько слабая сила, что какого то вменяемого эксперимента нельзя провести (именно по этому и решений каких то нет). Могут быть косвенные подтверждения, например если на коллайдере смогут "пропихнуть" частицу в другое измерение, это косвенно будет свидетельстовать о струнах. Но как и суперсиметрию, никаких дополнительных измерений и других ожидаемых открытий кроме хигса не нашли. Нужны больше мощности (но они тоже ничего не гарантируют).
>Есть ли у физики потолок? Что будет, когда мы достигнем его?
Пока потолка не видно. За 100 лет очень сильно поменялось представление о том как мир работает (в технологиях это очень заметно), сначала квантовая механика, теория полей, калибровочная инвариантность. Мы по прежнему не понимаем природу гравитации и кучу других вещей. Есть еще момент, что вселенной наплевать на людей. Другими словами, возможно у людей никого не получится понять истинную природу вещей, потому что инструмент, наш мозг, просто не предназначен для этого. Если же разобраться во всем можно, то достингуть потолка = быть богом.
>Теории Заговора
Вызваны низким интеллектом
>под давлением эмоционального груза ёбнуться головой?
Если есть генетические предрасположенности, и "ебнутся головой" = болезнь, то да - можно.
>На чём основываются предположения о многомерности реальности
Простыми словами - если измерений меньше 10 или 11 (это две разные теории, уже со временем) то струны просто не работают. Им "нужны" эти дополнительные измерения, если их убрать - математика становится бессмысленной. Да обязательно, потому что время это свойство самого пространства, из того что мы знаем на текущий день.
>Из чего состоит бозон Хиггса?
Ни из чего не состоит. Бозоны это переносчики взаимодействий, бозон хигса - это возмущение поля хигса. Из чего состоит фотон? Не из чего, это возмущение электромагнитного поля, которая то волна, то частица, то вообще в электрон превратилась на какие то 10-6 секунды, а потом опять стала фотоном.
Аноним 16/01/20 Чтв 02:49:45 49322067
>>493214
>потому что время это свойство самого пространства
Пространственного-временного континуума, как мне кажется, будет точнее. Они там типа как близнецы братья, не знаю кто для континуума более ценен, мы говорим время - привязываем пространство, пространство - добавляем время.
Аноним 16/01/20 Чтв 02:55:28 49322368
Аноним 16/01/20 Чтв 03:06:57 49322469
>>493223
>>пук
Флатуленция, как мне кажется, будет точнее.
Аноним 16/01/20 Чтв 04:32:22 49322570
>>493214
>в электрон превратилась на какие то 10-6 секунды, а потом опять стала фотоном.
Охуительные ночные открытия в саентаче.
Аноним 16/01/20 Чтв 04:49:46 49322671
>>493094
Так, дальше что? Тела они свои из чего строят, кислорода?
Аноним 16/01/20 Чтв 05:02:00 49323072
>>493226
Быстрорастущие сорта выводят, в том числе гм, например увеличивая выработку соответствующих фитогормонов. Деревья, кустарники, овощи, вот это все.
Аноним 16/01/20 Чтв 05:07:28 49323173
>>493225
Ну так-то он полуправ, в электрон-позитрон, в процессе столкновения (и рассеивания) фотонов друг на друге.
Аноним 16/01/20 Чтв 05:11:09 49323274
>>492883 (OP)
Так кто-нибудь может нормально ответить схуяли считается что свет квантовый?
Аноним 16/01/20 Чтв 05:18:02 49323375
>>493232
Потому что проявляет квантовые свойства.
Аноним 16/01/20 Чтв 05:22:40 49323476
>>493226
>Так, дальше что?
А дальше вспоминаем курс школьной биологии. Все зеленые растения в процессе фотосинтеза выделяют кислород и поглощают углекислый газ, а в процессе дыхания поглощают кислород и выделяют углекислый газ. Процесс фотосинтеза проходит только на свету, поэтому растения днем выделяют кислород, а ночью они дышат кислородом и выделяют углекислый газ.
>Тела они свои из чего строят
Строят они их из углерода, кислорода, водорода и некоторого количества других элементов. Так же для жизни растения нужна энергия получаемая в процессе окисления углеводов, образовавшихся в процессе фотосинтеза.

С6Н12О6 + 6О2 = 6СО2 + 6Н2О + энергия
Аноним 16/01/20 Чтв 05:26:31 49323577
>>493233
Херовый пустой ответ.

Какие?
Аноним 16/01/20 Чтв 05:33:36 49323678
>>493235
Каков вопрос.
Например фотоны могут и интерферировать как волна, и рассеиваться как частица.
Аноним 16/01/20 Чтв 05:36:54 49323779
>>493232
>схуяли считается что свет квантовый?
С того, что выяснили, что свет это волны в электромагнитном поле с частотой от 400 до 800 ТГц, а теория электромагнитного поля наиболее разработана в квантовой электродинамике, в основе которой лежит представление о том, что электромагнитное поле обладает дискретными свойствами, носителями которых являются кванты электромагнитного поля.
Аноним 16/01/20 Чтв 05:44:34 49323880
>>493237
Не ответ.
>>493236
Не уверен насколько это ответ. Вроде не ответ.

Вопрос нормальный. Какое проявление физическое доказывает что свет квантовый?
Аноним 16/01/20 Чтв 05:47:01 49323981
>>493238
>Какое проявление физическое доказывает что свет квантовый?
Например
Ф О Т О Э Ф Ф Е К Т
О
Т
О
Э
Ф
Ф
Е
К
Т
Аноним 16/01/20 Чтв 05:57:55 49324182
Анон, не корми эту зелень. >>493238
Аноним 16/01/20 Чтв 06:00:16 49324383
>>493241
Хочу и буду. Хуле, в 6 утра тут особо выбора нет.
Аноним 16/01/20 Чтв 06:06:53 49324484
Аноним 16/01/20 Чтв 06:08:08 49324585
>>493241
Ты просто слишком тупой. Не можешь дать ответ на вопрос из-за тупости поэтому и рвёшься.
Аноним 16/01/20 Чтв 06:08:57 49324686
>>493244
Именно то, что написано.
Аноним 16/01/20 Чтв 06:09:51 49324787
>>493246
Не слишком плохо разбираешься в этой теме чтобы дать ответ.
Аноним 16/01/20 Чтв 06:10:17 49324888
Аноним 16/01/20 Чтв 06:13:46 49325089
>>493247
Плохому танцору... Посмотри свои вопросы, прежде чем удивляться ответам.
Аноним 16/01/20 Чтв 07:23:45 49325190
Почему после нескольких заправок зажигалка начинает хуже гореть?
Аноним 16/01/20 Чтв 07:35:15 49325391
>>493250
Не рвись.
У тебя из-за тупости включается защитная реакция психики, не ыл бы таким тупым-замечал бы это и не позволял себе такого поведения.
Аноним 16/01/20 Чтв 07:55:42 49325492
>>493251
Наверно, уменьшается давление заправленного газа.
Аноним 16/01/20 Чтв 08:29:32 49325593
>>493253
Не проецируй, я меланхоличен, как слон. Это ты тут вокруг и лаешь, и визжишь, и рвешься.
Аноним 16/01/20 Чтв 08:32:41 49325894
>>493254
Визуально жидкий газ (если стравить воздух и дозаправить) на том же уровне. Может, конечно, у нас тупо газ весь в баллончиках второй свежести.
Аноним 16/01/20 Чтв 09:23:18 49326395
>>493214
>гравитация на столько слабая сила, что какого то вменяемого эксперимента нельзя провести
Зато можно наблюдать.
ru.wikipedia.org/wiki/Детектор_гравитационных_волн
Аноним 16/01/20 Чтв 09:27:01 49326496
>>493263
>Зато можно наблюдать.
Нет.
>ru.wikipedia.org/wiki/Детектор_гравитационных_волн
Ну какбэ речь о квантовой гравитации, так что уноси откуда принес.
Аноним 16/01/20 Чтв 09:29:32 49326597
>>493251
>начинает хуже гореть
Что значит хуже? Пламя слабее? Клапан дешевый, вот давление газа на выходе и падает.
Аноним 16/01/20 Чтв 09:31:49 49326998
>>493265
>Пламя слабее?
Скорее неравномерно горит (зажигалка аля горелочка), неравномерно, иногда даже плюется, особенно хорошо разогреваясь. Но и на зажигалке для плиты наблюдал, пламя тоже "плюется" и даже иногда его срывает и гаснет.
Аноним 16/01/20 Чтв 09:34:40 49327199
>>493269
>Скорее неравномерно горит
Проблемы с выпускным клапаном. Дешевая китайская зажигалка, дешевый клапан.
Аноним 16/01/20 Чтв 09:36:17 493273100
>>493271
Добра тебе. Полезная в хозяйстве, но то пьезик дохнет, то горит жопно. Буду продолжать жрать кактус, значит.
Аноним 16/01/20 Чтв 09:56:01 493283101
>>493258
Тут возможно и в газе дело, и в его объеме. Да и сама зажигалка со временем портится.
Аноним 16/01/20 Чтв 10:51:32 493286102
>>493273
>Буду продолжать жрать кактус, значит.
Купи бензиновую зажигалку и не майся дурью.
Аноним 16/01/20 Чтв 11:08:24 493289103
largemem400.jpg (96Кб, 480x480)
480x480
>>493286
>бензиновую зажигалку
Есть, неудобная, пованивает, пламя холодное и коптит. Горелочка намного удобнее. Пикрил качество совсем говно, до этого была заметно лучше (больше, сопло с керамикой), прослужила года 4, но больше в продаже таких не видел.
Если отойти от оффтопов, какой механизм "плавления" мезонов в КГП? Что именно происходит и почему это необычно?
Аноним 16/01/20 Чтв 11:16:55 493290104
>>493234
>растения днем выделяют кислород, а ночью они дышат кислородом и выделяют углекислый газ.

Примерное соотношение(очень примерное) выделенного кислорода к поглощенному кислороду какое?
Аноним 16/01/20 Чтв 11:25:05 493291105
>>493043
В теории - да. Скопировать строение живого мозга и уменьшить размер элементов.

На практике (сейчас) - абсолютно нет. Пока совершенно не понятно как мозг работает. Вот на днях, например, открыли новый способ взаимодействия нейронов между собой, это увеличивает продуктивность обмена информацией между нейронами в разы.

На практике (через тысячу лет непрерывного прогресса) - скорее всего да.
Аноним 16/01/20 Чтв 11:35:23 493293106
>>493290
>Примерное соотношение
Будет зависеть от освещения, конкретного растения и конкретного варианта фотосинтеза (C4 и CAM минимизируют дыхание, там значительно больше кислорода). Для C3 фотосинтеза вроде как 1:1 в худшем случае (опять же от освещения), будет гонять туда-сюда глюкозу, в световой и темновой фазе.
Аноним 16/01/20 Чтв 11:36:53 493294107
Аноним 16/01/20 Чтв 11:37:56 493295108
>>493033
Есть. Нахуй писать если не знаешь? Дохуя хейтер рашки, что прям сил нет, как хочется унизиться?
Аноним 16/01/20 Чтв 11:40:03 493296109
>>493293
>плюс-минус 1:1
То есть продукция избыточного кислорода лесами это миф, получается? Если выдают, то ннмного? А какжи "легкие планеты"?
Аноним 16/01/20 Чтв 11:41:05 493297110
>>493296
Ты ответил на ту часть, которую смог понять, да?
Аноним 16/01/20 Чтв 11:42:58 493298111
>>493297
>>493297
Я задал уточняющий вопрос, так как не все понял. Это тобой порицается?
В чем твоя реальная проблема?
Аноним 16/01/20 Чтв 11:44:24 493299112
>>493296

Впрочем, я перечитал и все понял. Спасибо за внимание
Аноним 16/01/20 Чтв 11:44:43 493300113
>>493296
А вообще миф, да. Легкие планеты - водоросли. Доля высших растений невелика.
>>493298
>Я задал уточняющий вопрос, так как нихуя не понял.
Пофиксил, не благодари.
> Это тобой порицается?
Да.
>В чем твоя реальная проблема?
В том, что ты тупее школьника. Ну и тащем-то это твоя проблема, не моя.
Аноним 16/01/20 Чтв 16:36:00 493337114
>>493289
>какой механизм "плавления" мезонов в КГП? Что именно происходит и почему это необычно?
bump
Аноним 16/01/20 Чтв 19:34:49 493349115
Не совсем по теме, но насколько высока вероятность, что Rosseta@home от BOINC и другие ответвления используют полученную мощность для научных целей, а не, например, майнинга хуйни? И откуда форс взялся с автономной капсулой? Сколько форсили его, на протяжении лет шести не могу понять это.
Аноним 16/01/20 Чтв 19:45:55 493350116
>>493349
А про форс гарика куколда харламова почему с научной точки хрения не задумываешься?
А о печи-булероне зимой жарко?? А о доме из контейнеров? А о землянке? А о землянке из зарытых контейнеров???
Stirling !!f2XR348pxA 16/01/20 Чтв 20:55:40 493354117
Отвечает ли теория этногенеза Гумилева критерию Поппера?
Аноним 16/01/20 Чтв 21:03:10 493357118
>>493354
Не читал, но осуждаю
Аноним 16/01/20 Чтв 21:45:20 493361119
Когда говорят "компьютер", то понятно, что имеется в виду. А что будет значить "не компьютер"?
Аноним 17/01/20 Птн 01:52:57 493389120
>>493354
А она не полна. Не обнаружены пока способы природного воздействия, которые (якобы) продуцируют массовое появление гумилевских уберменьшей. Если будет кандидат на источник таких воздействий, можно будет проверить его поппер
Аноним 17/01/20 Птн 01:54:41 493390121
>>493361
Множество исключающее компьютеры и включающее все остальное

какие наркотики ты сегодня принял?
Аноним 17/01/20 Птн 02:24:19 493392122
Почему физики считают, что в мире больше чем четыре измерения?
Аноним 17/01/20 Птн 02:27:10 493394123
>>493392
Ты неправильный вопрос задал. Измерения, это чисто математическая конструкция для моделей. "В мире" никаких измерений нет.
Аноним 17/01/20 Птн 02:28:15 493395124
>>493392
Струнщики/бранщики? Потому что они больше математики. В какой-то момент показалось, что элементарные частицы можно объяснить плюс-минус той же математикой, которая описывает колебания струны. Это так было красиво, даже изящно. Но с тех пор им пришлось ввести от 11 до 27 вроде доп. измерений, чтобы их математика согласовывалась, и теперь это жуткий монстр, который еще и не подтверждается никак.
Аноним 17/01/20 Птн 02:58:40 493398125
>>493394
В моём их 3, говори за себя.
Аноним 17/01/20 Птн 03:07:03 493399126
>>493398
Твоя психика воспринимает что их 3. Дай я тебе ЛСД, ты мне и про 4 расскажешь. Просто взаимодействия, лучше всего описываются в три+время. Нет ничего удивительного, что для убегания от тигра и лазания по деревьям, твой мозг тебе дал трехмерное представление. Представь, что вместо цвета предметов, ты бы мог воспринимать их как "дальность" в четвертом измерении, ты бы мне сейчас загонял за 4 в твоём мире?
Аноним 17/01/20 Птн 03:22:44 493400127
>>493399
У тебя шизоидные маняфантазии, если бы у цветов была дальность, и мы этого не видели бы, это видели бы приборы.
А если она никак не проявляется, то её и нет.

А ты ведёшь себя как верунская тупая пизда. "Докажите мине што бога нет!!1"
Так его блять нет, что тут доказывать.
Аноним 17/01/20 Птн 03:22:59 493401128
>>493398
Космологи говорят, что 11 измерений.
Аноним 17/01/20 Птн 03:30:43 493404129
>>493398
>В моём их 3
Доказать сможешь?
Аноним 17/01/20 Птн 03:31:41 493405130
>>493404
Конечно. Больше-не нужно.
Аноним 17/01/20 Птн 03:32:17 493406131
>>493399
>Представь, что вместо цвета предметов, ты бы мог воспринимать их как "дальность" в четвертом измерении
Можно представить, что поля, которыми заполнено все пространство, например электромагнитное это тоже измерение которое мы воспринимаем как поле.
Аноним 17/01/20 Птн 03:32:47 493407132
>>493401
1)И чо?
2)Ты точно понял что они говорят? Или у них в моделе для расчётов 11 измерений?
Аноним 17/01/20 Птн 03:32:50 493408133
>>493405
>Больше-не нужно.
И как ты докажешь, что ненужно?
Аноним 17/01/20 Птн 03:33:52 493409134
>>493400
>это видели бы приборы
Они и видят. Длина волны. Мы её воспринимаем как цвет. Ты хоть примерно понимаешь, почему какой-то предмет ты видишь дальше а какой-то ближе? Не думал, что просто один предмет электромагнитно взаимодействует на твою сетчатку одним образом, а другой просто иначе? И в чём принципиальная разница между длиной волны и расстоянием? В физических формулах и волновое число и координата, входят в аргумент для периодической функции, описывающих электромагнитное поле. И хуле? Ты свой субъективный опыт то не приноси. А то я так скажу, что и спина нет. Так как ты его не видишь.
Аноним 17/01/20 Птн 03:34:25 493410135
>>493408
А мне тебе ничего и не нужно доказывать, тупая вернуская пиздень.
Аноним 17/01/20 Птн 03:43:03 493411136
>>493410
>не нужно
Лол, просто ты не можешь это сделать. Тебе ума для этого не хватает.
Аноним 17/01/20 Птн 03:43:55 493412137
>>493409
Ты сейчас ему мозг сломаешь, осторожней.
Аноним 17/01/20 Птн 03:48:46 493413138
>>493409
>>493408
Ебать вы тут. Их три (четыре - плюс время), потому что мы задаем положение любой точки системой трех линейных независимых векторов. А не больше (по крайней не свернутых), потому как половина физических законов работают именно так, как работают только с тремя измерениями. Если бы их было больше - они бы работали иначе.
Таблетки выпейте и идите поспите.
Аноним 17/01/20 Птн 03:50:17 493414139
>>493413
>мы задаем положение любой точки системой трех линейных независимых векторов
Куда ты потерял спиноры?
Аноним 17/01/20 Птн 03:52:23 493416140
>>493414
Один сломал - второй потерял.
Аноним 17/01/20 Птн 03:54:30 493417141
>>493413
>Их три
Как докажешь?
>потому что мы задаем положение любой точки системой трех линейных независимых векторов
А кто нам запрещает задавать пятью?
>Если бы их было больше - они бы работали иначе
Как именно? Мне кажется, что ты не сможешь объяснить. Разочаруй меня и объясни.
Аноним 17/01/20 Птн 03:55:21 493418142
почему по ночам многомерные шизики так активизируются?
Аноним 17/01/20 Птн 03:59:20 493419143
>>493417
>Как докажешь?
>задаем положение любой точки системой трех линейно независимых векторов
This
>А кто нам запрещает задавать пятью?
Никто. Какие последние два, и с какой целью ими что-то задавать?
>Как именно?
Закон обратных квадратов.
>Мне кажется
Тебе уже мерещится.
Аноним 17/01/20 Птн 04:24:24 493422144
>>493418
Потому что не могут осилить курс линейной алгебры.
Аноним 17/01/20 Птн 04:32:22 493423145
>>493409
Говнопиздёж, на понт меня взять хочешь, хуй тебе.
Аноним 17/01/20 Птн 05:20:10 493424146
>>493419
>адаем положение любой точки системой трех линейно независимых векторов
А кто мешает делать это в произвольном многообразии с произвольной метрикой? Кто сказал, что векторов должно быть 3?
Аноним 17/01/20 Птн 05:24:20 493425147
>>493423
Ты видимо недостаточно образован чтобы понять и оценить всю красоту тех простых аргументов, что привел этот анон.
Аноним 17/01/20 Птн 06:42:52 493430148
>>492883 (OP)
Я изобрел вечный двигатель на магнитах. Куда обращаться чтобы запатентовать?
Аноним 17/01/20 Птн 09:34:08 493438149
Аноним 17/01/20 Птн 09:58:03 493439150
>>493430
>Куда обращаться чтобы запатентовать?
вечного ничего нет. твой генератор остановится, когда NdFeB поетряют намагниченность, что, конечно, случиться горзадо медленнее, чем для обычного железа, но тем не менее произойдет.
лучше никуда. не обращаться. или убьют, или невыездным сделают.
мимо изобрел его 15 лет назад
Аноним 17/01/20 Птн 12:09:42 493443151
>>493430
>Я изобрел вечный двигатель
>Куда обращаться?
ДС, Загородное шоссе, д. 2.
Аноним 17/01/20 Птн 14:22:32 493457152
Аноним 17/01/20 Птн 22:31:06 493522153
>>492883 (OP)
Вопрос тупой, поэтому задам его здесь.
Что нас ведёт к ТОРЖЕСТВУ КОММУНИЗМА?
Аноним 17/01/20 Птн 22:57:08 493524154
Аноним 17/01/20 Птн 23:13:53 493527155
>>493524
Социализм - годен лишь отчасти, но это хуйня, так как он неполноценен и вырастает из капитализма,
и вообще его победил, нахуй - капитализм в холодной войне!
А вот настоящий коммунизм - накроет пиздой весь глобальный капитализм,
вместе с низкими интересами капиталистоблядков ссаных,
в результате широкомасштабной коммунистической революции, няшной, заебатой и целенаправленной.
Аноним 17/01/20 Птн 23:37:53 493533156
>>493522
Как там, как там в гимне сверхдержавы великой было...
Щас вспомню... Во! Может быть партия Ленина и сила народная?
Надо, короче, погуглить чё там Ленин писал,
показать народу,
а потом просто и методично - продавить всё это силой Ньютона.
Аноним 17/01/20 Птн 23:56:39 493536157
Аноним 18/01/20 Суб 00:02:09 493537158
>>492993
Может потому что он не назывался - Будущее Человечества?
https://ibaka.ru/ReYYxDGIYyw
>>492994
Кстати да, последний пост был 12 января, не мог же он за 6 дней уплыть на 10 страницу и удалиться, тут не настолько активный постинг.
Аноним 18/01/20 Суб 00:05:38 493538159
image.png (22Кб, 600x184)
600x184
>>493537
Ах, сорян, назывался.
Аноним 18/01/20 Суб 00:48:24 493544160
Аноним 18/01/20 Суб 00:58:18 493545161
>>493059
>формация

Что у тебя? Формация? КОНФОРМАЦИЯ!
Это что-то вроде конформной формации. У капитализма такое свойство есть.
Выражается она в том, что, со временем, капиталисты всё мелочнее и мелочнее становятся,
но структура капитализма, как формации - сохраняется.
Аноним 18/01/20 Суб 01:02:18 493546162
image.png (3504Кб, 1920x1278)
1920x1278
>>493544
Лолблять. Если бы эта подкова не схлопывалась как клешня - другой базар бы был.
Аноним 18/01/20 Суб 01:24:55 493552163
>>492883 (OP)
Анон, про понятия мне разотри.
Аноним 18/01/20 Суб 14:00:21 493642164
Аноним 18/01/20 Суб 14:07:31 493644165
Аноним 18/01/20 Суб 17:01:45 493660166
>>493644
Нет, ты. А я - слегка приподнятый.
Аноним 18/01/20 Суб 18:24:34 493678167
После окончания ВУЗа собираюсь пойти работать в НИИ (астрофизика). Опыта вообще нет, но амбиции есть. В целом, как там обстановка? Много выдрачиваться надо? Вот не знаю, но у меня дикая тяга к исследованию, обработке и изучению, поэтому и появилось желание. Можете за НИИ пояснить?
Аноним 18/01/20 Суб 19:08:32 493688168
>>493678
Искаит пхд на западе.
18/01/20 Суб 21:25:06 493703169
>>492883 (OP)
Где купить грамм 100-200 оптического отбеливателя? Что наши барыги, что на алибабе готовы поставлять десятками и сотнями тонн для производства. Охуеть блядь, я хочу в банку лака добавлять для УФ контроля качества нанесения, и то не всегда, нахуя мне вагонами. В химии вообще бывает "опытное производство", по аналогии с машиностроительным?
Аноним 19/01/20 Вск 02:23:36 493721170
>>493703
В магазине химтоваров.
Аноним 19/01/20 Вск 02:30:30 493722171
Почему когда дует ветер мне холодно? Разве молекулы воздуха сталкиваясь с моей кожей, не должны передавать ей кинетическую энергию и тем самым согревать?
19/01/20 Вск 02:34:31 493725172
>>493722
>Разве молекулы воздуха сталкиваясь с моей кожей, не должны передавать ей кинетическую энергию

Теплопередача — физический процесс передачи тепловой энергии от более горячего тела к менее горячему.
Аноним 19/01/20 Вск 02:42:52 493726173
>>493349
У меня как-то была мыслишка на этот счёт. Подозреваю, что это доселе религиозный шизик, почитал эгоистичный ген и соответственно переформатировал свой ебанизм.
Возможно он начал считать, что гены это что-то вроде богов, которым нужно подражать и служить. Мол раз гены кодируют белки, которые становятся резервуаром для их сохранения и распространения, то нужно всячески продвигать кодинг белков и создание непрошибаемых капсул. То есть типа вести себя как ген.
Аноним 19/01/20 Вск 02:47:25 493727174
>>493725
Но разве по итогу температура это не кинетическая энергия атомов/молекул?

Тип вот если мы возьмём бак, начнём под ебанистическим давлением запузыривать в него воздух, то он нагреется. От того что молекулы хуярят по стенкам всё с большей скоростью и соответственно силой.
Аноним 19/01/20 Вск 02:56:41 493729175
>>493727
>Но разве по итогу
По итогу твое тело теплее воздуха, значит тепло передается от твоего тела воздуху. Ты тупой что ли?
19/01/20 Вск 03:04:57 493730176
image.png (1462Кб, 720x1280)
720x1280
>>493727
>температура это не кинетическая энергия атомов/молекул?
Нет. Температура это если и кинетическая энергия, то только для идеального газа, и то, только беспорядочная кинетическая энергия, которая не имеет направления. Температура, это статистическая величина. В некоторых случаях она и отрицательная может быть.
>если мы возьмём бак, начнём под ебанистическим давлением запузыривать в него воздух, то он нагреется
Он нагреется, от того, что ты потратишь работу по запихиванию воздуха в бак, а эта работа, по сохранению энергии перейдет в тепло.
Аноним 19/01/20 Вск 03:15:18 493731177
>>493729
>Ты тупой что ли?
Эээ... нууу... я эта... того... ну... ну как бы... как бы... типа... это же... ну то есть... эта же... же ну... как бы... тупых... тупых вопро... тупых вопросов тред... да..? Я... просто... как бы... думал... подумал... что... ну... как бы спро спросить... спросить можно же тут... как бы... этого? Ну... если у тебя это... то есть... ту тупой если... этот самый... как его... вопрос... во, вопрос то как бы... значит... здесь можно же... можно же здесь тогда да? Ну в смысле... тупой вопрос вот есть когда... когда вот у тебя есть... такой вот вопрос, то это же... ну... значит можно же... задавать? Ну, как бы, здесь вот в смысле... я просто не понял сразу... точнее понял что тут как бы это... ну можно как бы... потому что... эта... тут эта... ну как бы тред такой... вот я и решил что можно тут эта... понимаешь, как бы ну... да?
Аноним 19/01/20 Вск 03:22:16 493732178
>>493730
Отрицательной? Разве это не зависит исключительно от выбранной нами шкалы?
Вроде же по Кельвину не бывает температуры ниже нуля.

Но ведь я с баком непосредственно не взаимодействую. Только с его частью. А нагревается он весь. И от чего он собственно нагревается? Разве не от молекул воздуха, которые изнутри по нему хуярят?
19/01/20 Вск 03:24:21 493733179
>>493732
>Вроде же по Кельвину не бывает температуры ниже нуля.
В редких случаях может быть. В очень редких, там особые системы.
>Но ведь я с баком непосредственно не взаимодействую. Только с его частью. А нагревается он весь. И от чего он собственно нагревается? Разве не от молекул воздуха, которые изнутри по нему хуярят?
Сначала нагреется воздух в баке, который отдаст тепло баку просто.
Аноним 19/01/20 Вск 03:36:40 493737180
>>493733
Может ли существовать тепло там, где нет вещества? Не инфракрасное излучение, а вот чиста тепло?

И почему тогда вещи плавятся? Ну вот типа, как я понимаю почему они плавятся:
Есть твёрдое вещество. Допустим вот метал и в нём кристаллическая решётка. Атомы в нём очень лениво двигаются, чуть ли не на месте стоят. Тут мы повышаем температуру, атомы живенько начинают двигаться, да так живенько что аж из штанов выпрыгивают, по итогу кристаллическая решётка распадается и метал плавится.
И вот как бы основываясь на этом я пришёл к выводу, что тепло это и есть кинетическая энергия атомов. И что как бы без атомов понятие тепла просто не имеет смысла.
Аноним 19/01/20 Вск 03:41:43 493738181
>>493737
>Не инфракрасное излучение, а вот чиста тепло?
А что ты принимаешь под "чиста тепло"?
Аноним 19/01/20 Вск 03:52:13 493739182
>>493738
Признаться, под чиста теплом, я понимаю кинетическую энергию атомов. Основываясь на моём понимании того, отчего металлы и не только плавятся.
То есть, в меру моего непонимания, теплота это что-то намертво привязанное к веществу и без него просто имеющего смысла.
Аноним 19/01/20 Вск 04:07:52 493740183
магнетар.jpg (62Кб, 720x720)
720x720
Слушайте, а вот магнетары. У них вот магнитное поле по 100,000,000,000 тесла.

Что произойдёт со мною, если я окажусь недалеко от такого вот магнетара? Допустим в десятке километров? Будет ли магнитное поле воздействовать на моё тело? И если да, то как? Что именно со мною произойдёт, если я окажусь в таком сильном магнитном поле?
Аноним 19/01/20 Вск 04:13:37 493741184
>>493739
>То есть, в меру моего непонимания, теплота это что-то намертво привязанное к веществу
А как теплота передается от вещества к веществу?
Аноним 19/01/20 Вск 04:31:51 493742185
>>493741
Либо непосредственно, один атом ударяет другой и тем самым передаёт ему кинетическую энергию, или шатается кристаллическая решётка а с ней и все атомы до которых доходит шатание по её рёбрам; либо посредством инфракрасного излучения.
Оно [излучение] вроде как... заставляет атомы дрожать сильнее. Я затрудняюсь объяснить как и почему. Возможно атомы, вроде бы как, "плавают" в неком поле и "вибрация" этого поля передаёт кинетическую энергию атомам. Как, ну, как поплавок плавает в воде, но если рядом с ним начать бить веслом по воде, то он через воду получит кинетическую энергию и начнёт колыхаться.
Вот такая вот примерно картина у меня в голове.
Аноним 19/01/20 Вск 05:26:47 493743186
Как вырастить большой кубический кристалл хлорида натрия без нитки или лески внутри?
Аноним 19/01/20 Вск 10:10:27 493754187
Аноним 19/01/20 Вск 12:51:51 493776188
>>493740
У нейтронных звезд очень мощные приливные силы, так что тебя распидорасит на атомы.
Попутно нейтронные звезды горячие, светят в рентгене, а вблизи из-за магнитного поля мощные потоки частиц, причем посильнее чем в коллайдерах. В любом случае ты умрешь от приливных сил или радиации, прежде чем войдешь в область сильного магнитного поля.
Аноним 19/01/20 Вск 13:35:17 493784189
>>493740
Тело является проводником электричества.
А при движении проводника в магнитном поле -
происходит электромагнитная индукция:
https://www.youtube.com/watch?v=xZjBqwXxiRw
Аноним 19/01/20 Вск 13:47:18 493788190
>>493754
>Как-то так
Он на леску вешает затравку.
Аноним 19/01/20 Вск 13:48:55 493789191
>>493788
Так это и есть кусок кристалла. Получается он упариванием.
Аноним 20/01/20 Пнд 01:10:39 493882192
>>493741
Окей, твои наводящие вопросы заставили меня задуматься и переформатировать свою картину мира.
То есть если моё представление о том, как действует инфракрасное излучение в хотя бы каком-то приближении верно, то получается, что теплота это не кинетическая энергия атомов. Она лишь проявляется придавая кинетическую энергию атомам. А сама по себе теплота, это и есть инфракрасное излучение. Так?
Аноним 20/01/20 Пнд 01:15:48 493884193
>>493776
>>493784
Спасибо за ответы.

Правильно ли я понимаю, что если абстрагироваться от приливных сил, излучений и потока частиц, если оценить как будет воздействовать на меня исключительно магнитное поле такой силы, то выходит, что меня банально ёбнет током так мощно, что моё тело испепелит и разметает в углеродную пыль?
Аноним 20/01/20 Пнд 01:31:27 493886194
>>493882
>То есть если моё представление о том, как действует инфракрасное излучение в хотя бы каком-то приближении верно
А почему только инфракрасное? Нагреть можно и радиоволнами и видимым светом.
>А сама по себе теплота, это и есть инфракрасное излучение. Так?
Смотри. Вот у тебя есть мощный твердотельный лазер работающий в видимом свете, ты же представляешь как он устроен? Ты шарахнул из него по Ваське. Васька нагрелся?
Если Васька нагрелся, то как видимый свет его нагрел? И откуда Ваське передалось такое тепло, если в Ваське уже дымящаяся дырка, а кристалл в лазере чутка лишь нагрелся?
Аноним 20/01/20 Пнд 01:34:02 493887195
>>493884
Тушка человека довольно хуевый проводник. Весь создаваемый ток индукции будет уходить в тепло, так что заряд любви не почувствуешь, зато сварит изнутри. Впрочем тушка диамагнетик, так что ее будет выталкивать из области сильного магнитного поля.
Впрочем у магнетаров настолько сильное поле, что оно разрушает химические связи и даже атом, за счет сильного возмущения орбиталей. Очень грубо говоря, ядро и электрон сами по себе магниты, а при сильных внешних полях, локально превосходящих их, становиться невозможны привычного атома - электрон и ядра получаются не связанными. В любом случае тебя и так распидорасит.

20/01/20 Пнд 03:29:30 493894196
>>493887
Все атомы в твоем теле превратятся в неведомую цилиндрическую ебень.
Аноним 20/01/20 Пнд 17:20:09 493999197
В теории ведь можно размотать ЧД до меньшей итоговой массы, столкнув две ЧД на большой скорости по касательной? Теоретически для цивилизации, для которой не проблема ПВК и разделение/слияние звезд (кроме конечности количества исходных звезд/карликов). Например вращающиеся ЧД магнитным полем разгоняют, сливают, ЧД потеряют массу, и цивилизация получит как бонус еще более быстро вращающуюся дыру (а дальше дело техники, клепает нужного размера дыру, дальше строит вокруг генератор и живет на запасенной энергии когда уже протухли все звезды)? Или я где-то фатально ошибся в рассуждениях?
Аноним 20/01/20 Пнд 17:30:59 494000198
>>493999
Да. Цивилизация не станет так сложно хуевертить, а просто зажжет новую звезду.
Аноним 20/01/20 Пнд 17:37:40 494002199
>>494000
А последняя потухла, карлики иссякли, что делает? Просто зажигает еще одну из карликов, чай не тупые, умеют не только в натуральные числа, будет -2 карлика в запасе?
Идет лекция матана в субботу, в аудитории только три студента, пять выходят, профессор думает: "Ну вот, еще два опоздавших придут и вообще никого не останется!".
Аноним 20/01/20 Пнд 21:54:13 494037200
>>494002
Остаётся ещё куча газовых гигантов которых можно столкнуть и получить звезду, но если ты уже умеешь двигать газовые гиганты, то ты просто будешь сжигать их водород без делания из них звёзд и без разбрасывания 99.99% энергии впустую в открытый космос.
Аноним 20/01/20 Пнд 22:18:06 494045201
Существует расхожее мнение, что в течении 7 лет все до последнего атома в теле человека обновляются. В связи с этим есть ряд вопросов:
1. Действительно ли это так, или это обычное натягивание совы на глобус?
2. Если это так, каким образом мы сохраняем более менее постоянный вид? Где сохраняется информация, каким образом эти атомы должны соедениться? Я догадываюсь, что за это отвечает ДНК, но не понимаю механизма
3. Почему, раз материал из которого изготовленно тело полностью обновляется, не устраняются дефекты, не отростают конечности, не регенируются органы? Какой смысл атомам собираться в поломаное тело?
Аноним 20/01/20 Пнд 22:26:32 494047202
>>494045
>3. Почему, раз материал из которого изготовленно тело полностью обновляется, не устраняются дефекты, не отростают конечности, не регенируются органы? Какой смысл атомам собираться в поломаное тело?
Предположу что они не знают что они собираются в поломанное тело, они же вообще думать не могут.

А форма сохраняется потому что они ПОСТЕПЕННО заменяются, т.е. один съебал, на его место встал новый, т.е. ничего не изменилось.

И это не поатомно происходит, а поклеточно.
Хотя вот с зубами хз, они не клеточные, вряд ли они заменяются. По крайней мере неживой слой.
И со скелетом хз.
Кароче всё что состоит не из клеток наверное не заменяется.
Аноним 20/01/20 Пнд 23:47:35 494062203
>>492883 (OP)
Учёные, так когда вы говорите ледник там последний был?
На территории европы и выше.
Аноним 21/01/20 Втр 00:18:18 494066204
Допустим, я попал в средневековье. С собой у меня есть ноутбук с полезной информацией, зарядка от него и десяточка в харизме, так что все будут слушаться меня и казнят только если я оскорблю матушку короля. Как мне зарядить ноутбук?
Местность на ваше усмотрение, так как по условию я могу приехать в любое место. Язык тоже знаю.
Аноним 21/01/20 Втр 00:29:25 494069205
>>494002
>что делает?
Делать новую Вселенную и перебираться туда.
Аноним 21/01/20 Втр 00:35:34 494072206
>>494062
Последняя ледниковая эпоха (вюрмское оледенение, вислинское оледенение, валдайское оледенение) — последняя из ледниковых эпох на территории Европы в рамках плейстоценового или четвертичного цикла ледниковых периодов.

Оледенение началось в Северной Европе около 110 тыс. лет тому назад и окончилось около 11,7 тыс. лет назад. Во время этой эпохи неоднократно происходило разрастание и сокращение ледниковых покровов. Максимум последнего оледенения, когда общий объём льда в ледниках был наибольшим, относится ко времени около 26,5—19 тыс. лет назад.
Аноним 21/01/20 Втр 00:36:10 494073207
>>494066
>Как мне зарядить ноутбук?
Никак.
Аноним 21/01/20 Втр 00:59:59 494085208
>>494072
>110 тыс. лет тому назад и окончилось около 11,7 тыс. лет назад.
Хуясе, насколько больше льда чем без льда.

А как же "на планете большую часть времени был равномерный субтропический климат" кто-то тут писал?
Аноним 21/01/20 Втр 01:01:09 494086209
>>494072
>Максимум последнего оледенения, когда общий объём льда в ледниках был наибольшим, относится ко времени около 26,5—19 тыс. лет назад.
И до куда они доходили? Беларусь и ниже была вся покрыта ледником? На украину он заезжал? А до чёрного моря/на чёрное море заезжал?
Аноним 21/01/20 Втр 01:17:15 494094210
>>494085
>А как же "на планете большую часть времени был равномерный субтропический климат" кто-то тут писал?

А сколько по-твоему лет существует Земля?
Аноним 21/01/20 Втр 01:23:43 494097211
>>494094
Так оледнение было не одно же. Тут одно длилось 100к лет, другие сколько длились?
Аноним 21/01/20 Втр 01:24:18 494098212
>>494086
В это время оледенение захватило большую часть Северной Америки, Скандинавский полуостров, север Европы и Восточно-Европейской равнины. Льдами были покрыты Альпы и Гималаи, южные оконечности Южной Америки и Австралии. Климат стал не только более холодным, но и более сухим, что привело к уменьшению площади лесов и опустыниванию многих регионов, таких как Южная Австралия.
Да, Беларусь тоже была под ледником.
Украина тоже, ледник доходил до верховьев Днепра.
Аноним 21/01/20 Втр 01:25:00 494099213
>>494097
>другие сколько длились?
По разному, но большую часть истории Земли на ней не было ледников.
Аноним 21/01/20 Втр 02:07:31 494129214
IMG202001210215[...].jpg (113Кб, 720x1280)
720x1280
>>494098
>южные оконечности Южной Америки и Австралии.
Хренасе, это что прям до сюда доходил?
Аноним 21/01/20 Втр 02:08:43 494131215
>>494099
Удевительно что земля-снежок оттаяла вообще.
Аноним 21/01/20 Втр 02:09:26 494132216
Аноним 21/01/20 Втр 03:19:31 494150217
Аноним 21/01/20 Втр 03:20:12 494151218
>>494129
>>494098
>Климат стал не только более холодным, но и более сухим, что привело к уменьшению площади лесов
О каких лесах тогда вообще речь? Где они были?
Аноним 21/01/20 Втр 03:20:50 494152219
>>494098
>Украина тоже, ледник доходил до верховьев Днепра.
>>494129
Но на этом пике нихера не верховья днепра, тут африка полностью накрытая, лол.
Аноним 21/01/20 Втр 03:51:47 494163220
>>494152
>тут африка полностью накрытая, лол.
При последнем оледенении большая часть Африки не была покрыта ледниками.
Аноним 21/01/20 Втр 04:15:25 494169221
>>494163
>При последнем оледенении большая часть Африки не была покрыта ледниками.
>>494098
>южные оконечности Южной Америки и Австралии.
А тут что тогда написано?
На пике я отметил южную оконечность Австралии.
Аноним 21/01/20 Втр 04:19:34 494171222
>>494169
>А тут что тогда написано?
Что последнее оледенение затонуло южные оконечности Южной Америки и Австралии.

Во время последнего оледенения, в Африке, кроме как на Килиманджаро и ряде других вершин, ледников не было.
Аноним 21/01/20 Втр 04:50:09 494173223
>>494171
Т.е. там просто температура холоднее стала или что?

Как-то не подходит под термин "оледенение".
Там был лёд, ледник, или как?
Аноним 21/01/20 Втр 05:01:27 494174224
>>494173
>Т.е. там просто температура холоднее стала или что?
Да. Упала температура и воздух стал суше, что привело к очередному уменьшению тропических лесов Африки и заменой их бушем. В прошлый раз такие явления привели к тому что предки людей вышли из лесов в буш, это привело к прямохождением, а в это (последнее) оледенение вероятно это привело к тому, что люди начали активно расселяться из Африки.

>Как-то не подходит под термин "оледенение".
Мы говорим о последнем оледенении, оно не закрыло ледниками всю планету, как в прошлых оледенениях.
Аноним 21/01/20 Втр 06:08:41 494179225
>>494174
Т.е. 15-20 тысяч лет назад на территории всей современной европы и до низа украины вообще не было никаких животных и лесов?
Аноним 21/01/20 Втр 06:36:49 494188226
DL5kx47UIAA7DFY.jpg (92Кб, 1200x721)
1200x721
>>494179
Держи карту покрытия ледником в максимум последнего оледенения (18 тысяч лет назад)
Аноним 21/01/20 Втр 06:51:34 494190227
>>494188
Так не много совсем, на беларусь не заходило в тот раз.

А писал до низа украины...
Аноним 21/01/20 Втр 06:54:29 494191228
>>494190
>А писал до низа украины
Ты карту-то посмотри внимательно.
Аноним 21/01/20 Втр 06:56:12 494192229
>>493409
> примерно понимаешь, почему какой-то предмет ты видишь дальше а какой-то ближе? Не думал, что просто один предмет электромагнитно взаимодействует на твою сетчатку одним образом, а другой просто иначе?
Не поэтому, а потому что зрачок на разных фокусировках дает разную четкость разных объектов
Аноним 21/01/20 Втр 07:26:44 494195230
>>494191
Ну, там написано серое это лёд, лёд только верху немного был.
Аноним 21/01/20 Втр 07:45:35 494198231
>>494192
Ты в курсе чем занимается оптика и что такое свет?
Аноним 21/01/20 Втр 09:14:20 494203232
>>494192
>Не поэтому, а потому что зрачок на разных фокусировках дает разную четкость разных объектов
Охуеть, тут у анона ЗУМ встроен нахуй! Я ебал.
Аноним 21/01/20 Втр 09:15:07 494204233
>>494192
Если было непонятно из предыдущего сообщения, восприятие дальности и чёткость/размытость это совершенно разные параметры.
Аноним 21/01/20 Втр 10:23:22 494206234
>>494045
Хорошие вопросы, тоже хотет ответ
Аноним 21/01/20 Втр 10:28:23 494207235
>>494066
Апельсины наверное проволокой в цепочку соединить, они вырабатывают немношк тока, но надо контролировать как-то силу тока и напряжение мультиметром, так что придется действовать так:
1) приходишь к королю Венеции или чего-нибудь еще продвинутого, говоришь: у меня есть инфа как продвинуть ваши технологии на столетия вперед за десятилетие (показываешь ноут, король в ахуе), но мне нужно дохрена щарящих подчиненных
2) тебе дают целую гору холопов
3) с ними развиваешь индустрию электроники, через 10 лет создаёте мультиметр
4) заряжаешь с должной силой тока и напряжением ноут
Аноним 21/01/20 Втр 10:34:08 494212236
Аноним 21/01/20 Втр 10:36:32 494215237
>>494203
Если ты на ближний объект сфокусировался, то дальний будет размыто выглядеть, вот так при неподвижной голове можно оценивать расстояние. При подвижной - еще и параллаксом относительно бэкграунда. При большом расстоянии до объекта - еще и снижением его контрастности и синей дымке от слоя атмосферы между ним и тобой.
Аноним 21/01/20 Втр 10:37:12 494216238
>>494203
И еще, глаза 2, параллакс можно поэтому оценить даже при неподвижной голове, еали объект достаточно близко
Аноним 21/01/20 Втр 10:37:29 494217239
Аноним 21/01/20 Втр 10:44:58 494219240
>>494207
Нужен уксус и металл более менее активный. И этого достаточно чтобы сделать батарейки простецкие. Подключая их последовательно, можно добиться любого напряжения.
P.S. Ты что, никогда не делал дома из монеток, бумаги, фольги и раствора соды батарейку?
Аноним 21/01/20 Втр 10:58:35 494223241
>>494215
>Если ты на ближний объект сфокусировался, то дальний будет размыто выглядеть, вот так при неподвижной голове можно оценивать расстояние.
Ты ебанат?
А если на дальний объект сфокусироваться, то БЛИЖНИЙ будет размыто выглядеть, оцени мне тут расстояние блядь.
Пиздец уровень /sci/
Аноним 21/01/20 Втр 11:20:22 494225242
>>494223
> если на дальний объект сфокусироваться, то БЛИЖНИЙ будет размыто выглядеть, оцени мне тут расстояние блядь.
Ты же чувствуешь, какой объект близко, а какой далеко, то есть чувствуешь, в какую сторону ты глаз перефокусируешь - ближе или дальше. Так что да, это позволяет оценить с некой вполне приличной точностью расстояние.
Аноним 21/01/20 Втр 11:24:58 494226243
>>494225
И вдобавок еще один фактор тут помогает - экспириенс. Ты знаешь многие типы объектов, ты видел подобные раньше. Например, ты знаешь размер мячика и размер стены, и будешь понимать, что если ты видишь мячик на фоне стены, то мячик ближе (к тому же по z-order’у мячик ближе - перекрывает изображение стены собой), и ты знаешь типичный размер мячика и типичный размер кирпичей в стенах.
Аноним 21/01/20 Втр 11:26:30 494227244
>>494226
И это, вкупе с параллаксом при движении камеры, позволяет даже в 2D-фильмах точно оценивать расстояния
Аноним 21/01/20 Втр 11:58:58 494231245
>>494225
>>494226
Не знаю насчёт рыбьих глаз, но мозг и понимание физики у тебя точно рыбье.
Аноним 21/01/20 Втр 12:37:36 494233246
>>494226
>>494225
Ты одноглазым родился что-ли? Или что? Или ты просто шестиклассник? Ты ебанутый? Ты хоть раз на фильм в 3D ходил, как думаешь, нахуй там очки?
Аноним 21/01/20 Втр 13:15:30 494236247
son-stereo-daun[...].jpg (115Кб, 813x500)
813x500
>>494225
Хуя тут острие науки. Вообще-то общеизвестно, что оценка расстояния работает хорошо только при наличии 2 глаз, догадываешься почему? Остальные механизмы тоже используются как вспомогательные после обучения, там сравнение со "знакомыми" (эталонными) объектами, сравнение угловых скоростей, аккомодация тоже. Но это все вспомогательно, основное - бинокулярное, у нас даже зрачки "сцепились", лишая панорамы, чтобы реализовать оценку расстояния.
Аноним 21/01/20 Втр 13:58:52 494239248
>>494231
Ты не пукай, а обоснуй
Аноним 21/01/20 Втр 14:01:35 494240249
>>494233
Дак я и про параллакс от второго глаза написал >>494216
Но при большой удаленности объекта, только параллакс при твоем движении и снижающая контрастность синящая атмосфера уже годятся для точного определения
Аноним 21/01/20 Втр 14:03:52 494241250
>>494236
> Вообще-то общеизвестно, что оценка расстояния работает хорошо только при наличии 2 глаз, догадываешься почему?
Хуйня полная, наоборот, одноглазый может даже играть в бейсбол
Аноним 21/01/20 Втр 14:05:01 494242251
>>494240
На Луне вот астронавты не могли оценить расстояние до гор нормально, из-за отсутствия атмосферы
Аноним 21/01/20 Втр 14:24:06 494247252
>>494207
> развиваешь индустрию электроники
И как это сделать в средневековье? Нужен подробный гайд.
Аноним 21/01/20 Втр 14:27:24 494248253
>>494241
Пикрил-то посмотри, лишнехромосомный.
Аноним 21/01/20 Втр 14:29:57 494250254
>>494247
Ну короче металлы у них есть, чтоб провода сделать, как электрогенератор на магните сделать, думаю, вы там с мужиками тоже сообразите, ну и всё, че еще надо-то?
Аноним 21/01/20 Втр 14:31:21 494251255
>>494248
Да я посмотрел, дурачок, но это не отменяет того, что даже одноглазый без проблем мяч может поймать и не наебывается, когда ходит везде
Аноним 21/01/20 Втр 14:37:41 494252256
>>494251
>Глухой может играть на пианино
>РРРЯЯЯ ушебляди не нужны
Может, безногий может плавать, безрукий рисовать, как это логически связано с основным механизмом оценки расстояния?
Аноним 21/01/20 Втр 14:42:25 494254257
1232176.jpg (97Кб, 624x471)
624x471
>>494250
Ковать провода пробовал? Изготовление проволоки, намагничевание магнита... Нет, батарейка гораздо более здравая идея, но с одним ноутом вангую сожженый ноут на одной из первых пяти попыток. Алсо сожженого анона на первой попытке прокинуть харизму.
Аноним 21/01/20 Втр 14:58:03 494260258
APS.jpg (48Кб, 512x490)
512x490
>>494236
Все правильно. На этом основан целый раздел геодезии - фотограмметрия, когда по стереопаре полученной с аэро/космической съемки, на фотограмметрической станции строится карта местности, со всеми высотами. Одноглазый игрок в бейсбол может обосраться, но никогда не сможет измерить третью координату только по одному снимку.
Аноним 21/01/20 Втр 15:14:56 494264259
>>494252

> безногий может плавать, безрукий рисовать
Не могут нормально. А одноглазый может вполне легко поймать мяч.
Аноним 21/01/20 Втр 15:16:44 494265260
>>494254
> намагничевание магнита
А если уже магнитную руду использовать? Или я бредю?
Аноним 21/01/20 Втр 15:21:17 494267261
>>494260
Я перечислил все способы просто, и этот тоже. И да, для близких объектов он хорош. Одноглазый Игрок видит движение мяча в динамике и двигаясь сам, а значит хорошо видя параллакс, увеличение размера мяча на подлете, перефокусируя глаз и зная, насколько сильно он его перефокусирует и в какую сторону (на приближение), возможно даже видя увеличение контраста при приближении мяча.
Аноним 21/01/20 Втр 15:28:39 494268262
>>494264
Даун еще и упертый. Легко не может, как глухой играть, аналогия совершенно оправданная. Ну или эксперимент поставь, если четырехглазый: одень первый раз новые очки и походи с одним открытым глазом и двумя. Сравни ощущения, когда сбил остальные настройки, как тебе будет с одним глазом и с двумя оценивать. Насколько это хуйня, раз ты такой из себя не верун в открытия, которым тыщу лет в обед.
>>494265
Бредишь. Точнее обе две проблемы навскидку самые значимые, проволоку тянуть тяжело, особенно длинную, а тут надо намотку делать во много витков, и изготовление магнитов, чем сильнее - тем лучше, а у тебя в лучшем случае слабый магнит из железки, придется сначала научить намагничивать и "изобрести" пару сплавов с железом. А намагничивать - надо электромагнит, видимо. По крайней мере не знаю других способов. Еще при ковке/плавке естественный магнетизм по пизде должен пойти.
Аноним 21/01/20 Втр 15:32:51 494269263
Tesseract.gif (490Кб, 256x256)
256x256
>>494267
Динамика - это один из способов повышать размерность. К примеру мы можем понять как выглядит движущийся тессеракт, в четырех измерениях, видя по сути лишь 2д картинку, но которая изменяется. Для неподвижного объекта это не сработает, так как одноглазый не чувствует глубину.
Аноним 21/01/20 Втр 18:50:41 494298264
>>494264
>одноглазый
>вполне легко поймать мяч
И гуглятся одноглазые игроки (все питчеры): Tom Sunkel (1937-1944), Whammy Douglas (1957-1957), Abe Alvarez (2004-2006).
>легко
Аноним 21/01/20 Втр 19:40:24 494300265
>>494298
Хули они так мало жили?))
Аноним 21/01/20 Втр 19:41:53 494301266
>>494300
MLB не MLP, суровый мир.
Дискурс Инкогнито 21/01/20 Втр 19:54:20 494305267
image.png (652Кб, 750x726)
750x726
В виду отсутствия в научном сообществе единой, общепринятой формулировки этого понятия, прошу дать ту дефиницию, которая на ваш взгляд кажется наиболее близкой к истине.

Ну или хот бы просто объясните своими словами.
Аноним 21/01/20 Втр 20:17:33 494308268
Аноним 21/01/20 Втр 20:20:27 494309269
Аноним 21/01/20 Втр 21:58:36 494314270
>>494308
Собрать простейший вольтметр. А откалибровать его можно по какому-нибудь стандартному электроду, который можно изготовить из нужных хим. реактивов. Они все доступны были уже тогда.
Аноним 21/01/20 Втр 23:39:36 494320271
17f12cce0d[1].jpg (138Кб, 605x807)
605x807
Аноним 22/01/20 Срд 00:39:48 494336272
>>494320
Напоминает медузу и моллюска
Аноним 22/01/20 Срд 00:45:00 494341273
Аноним 22/01/20 Срд 01:32:17 494350274
>>494336
>>494341
Бля, а как им написать туда без регистрации и без смс?
Знаю, что TOR и torchat - использует RSA: https://github.com/Artifact0/Garlic
Так вот, было бы охуенно, если бы приватным ключём RSA,
сгенерированным локально,
можно было бы подписать собщение некое от сервера,
и безо всякой регистрации - оставить коммент,
от юзера myuserblahblah.onion

Вот бы по всему интернету такую систему авторизации ввести.
Аноним 22/01/20 Срд 02:00:43 494359275
15793739366410.jpg (190Кб, 786x664)
786x664
15793809803010s.jpg (4Кб, 168x170)
168x170
Аноним 22/01/20 Срд 02:07:47 494365276
>>494314
Ну это пустые слова о том что и так понятно, ты еонкретику давай.
Аноним 22/01/20 Срд 02:32:49 494374277
>>494359
Рили, privkey можно было бы хранить в LocalStorage, а подписывать расширениями к браузеру,
вроде MetaMask (могущим подписывать транзакции Ethereum приватным ключём)
Аноним 22/01/20 Срд 03:06:43 494378278
>>494365
>Ну это пустые слова
Может это просто у тебя голова пустая?
мимо
Аноним 22/01/20 Срд 07:36:43 494385279
Поясните по хардкору, можно ли пить дистиллированную воду без добавок.
Просмотрел пару статей - везде бредовая аргументация типа "это безвредно потому, что нет никакого вреда, мы так скозали, она вымывает только шлаки, токсины и те минераллы, которые не используются, а полезные минераллы она не может вымывать! НЕ МОЖЕТ!!! ЭТО НИВАЗМОЖНА!!!". Было б круто получить нормальную статью ну или хоть своими словами.
Аноним 22/01/20 Срд 07:45:23 494386280
>>494385
Можно. Это просто вода, ну получишь меньше ионов кальция/магния, заодно меньше ионов трубия, если сравнивать с водопроводной. Не знаю, какого еще эффекта ты ожидаешь от воды.
Аноним 22/01/20 Срд 07:55:47 494387281
>>494386
Покормлю: главный аргумент против - это что она
>вымывает
вещества по причине своей особой чистоты.
Аноним 22/01/20 Срд 08:03:52 494388282
Нахуя бог создал вирусы?
Аноним 22/01/20 Срд 08:19:13 494390283
>>494387
Аргумент уровня "вакцины вызывают аутизм", мы немного не губка, чуток сложнее устроены, и не фильтруем воду телом напрямую. "Меньше поступает" неравно "больше вымывает". Если из всех источников будешь получать меньше кальция в усваиваемом виде ежедневно (или будет нарушена его усваиваемость) - может развиться хрупкость костей, например. Но я бы даже двачеру не посоветовал бы полагаться на воду, как на его основной источник.
>>494388
Был блестящим математиком, опередившим свое время. Хотя я не стал бы называть его богом, я люблю фон Неймана, но не настолько.
Аноним 22/01/20 Срд 09:22:16 494394284
>>494268
> одень первый раз новые очки и походи с одним открытым глазом и двумя. Сравни ощущения
Что за бред? Просто попробуй поймать мяч с одним глазом - у тебя получится я в тебя верю а на пианино играть не получится, если только по памяти движения воспроизводить.

Аноним 22/01/20 Срд 09:24:45 494395285
Аноним 22/01/20 Срд 09:26:09 494396286
>>494300
Tom Sunkel Died: April 6, 2002 (aged 89)
Аноним 22/01/20 Срд 09:30:33 494397287
image.jpeg (113Кб, 750x1000)
750x1000
>>494219
> Ты что, никогда не делал дома из монеток, бумаги, фольги и раствора соды батарейку?
Нет
Аноним 22/01/20 Срд 09:40:04 494398288
>>494394
И то, и это получится. Я могу приглушить струны на гитаре и играть по памяти ИЗИ. Ты вон пишешь связный набор слов, совершенно не используя мозг. Сможешь рассмотреть глубину на "фантомашке" (автостереограмме) одним глазом - выходи на связь снова.
Аноним 22/01/20 Срд 09:49:36 494399289
>>494398
> Сможешь рассмотреть глубину на "фантомашке" (автостереограмме) одним глазом - выходи на связь снова.
Я могу рассмотреть глубину в любом 2д-кинце даже, это не вызывает ни у кого проблем
Аноним 22/01/20 Срд 10:46:38 494410290
9Wcifu2fKdg.jpg (103Кб, 678x1000)
678x1000
Прошу прощения за косноязычность, я имбецил.
Если один магнит запихнуть в стеклянную банку, а затем другой сверху тем же полюсом, мы заставим второй магнит левитировать?
Откуда берётся сила противодействующая силе притяжения?
Если магниты не теряют свойств десятки лет, то откуда энергия для поддержания данной конструкции?
Аноним 22/01/20 Срд 10:53:32 494411291
art-of-3d-07.jpg (179Кб, 800x400)
800x400
>>494399
Ну так в 2-д кинце нет глубины, я тебя разочарую. Рассмотри в пикрил.
Аноним 22/01/20 Срд 10:58:35 494412292
>>494410
>Откуда берётся сила противодействующая
Из магнетизма. А не удивляет, что когда ты стоишь на земле, откуда-то берется эта сила противодействующая? Почему не проваливаешься к ядру?
>то откуда энергия для поддержания
Как ты посчитал, что для поддержания тратится энергия? Какую работу совершает магнит, лежа в банке (ну или зависнув)?
Аноним 22/01/20 Срд 11:30:19 494419293
>>494410
>Откуда берётся сила противодействующая силе притяжения?
А никто не знает. Сейчас наши знатоки начнут мямлить и фантазировать.
Аноним 22/01/20 Срд 11:30:56 494420294
>>494419
>Сейчас наши знатоки начнут мямлить и фантазировать.

вот! >>494412

Аноним 22/01/20 Срд 11:33:06 494423295
>>494419
>>494420
Ей, шизик, как там в том месте, где еще про магниты (магнитное поле и его влияние на заряженные частицы) еще не слышали? Таблетки принял?
Аноним 22/01/20 Срд 11:40:26 494424296
>>494410
Смотри, тут никто не знает что такое магнитное поле, откуда оно берется и как работает. А если спросить, то тебя назовут шизиком >>494423
Аноним 22/01/20 Срд 11:42:53 494425297
/sci/ пиздец помойка, всё хужее и хужее с каждым годом
Аноним 22/01/20 Срд 11:44:37 494427298
>>494424
>А никто не знает.
Шизик не знает = никто не знает.
>Сейчас наши знатоки начнут мямлить и фантазировать.
Спросить.
Пой, птичка, не стыдись.
Аноним 22/01/20 Срд 11:46:34 494428299
>>494427
Так я такой же даун как и ты, откуда мне знать что такое магнитное поле, откуда оно берется и как работает?
Аноним 22/01/20 Срд 11:49:53 494429300
>>494428
>я такой же
Ты похуже червя-пидора, в трех постах сменил три разных позиции, давай дальше в миссионерской. Сказать что-то хотел, али просто бедтрип?
Аноним 22/01/20 Срд 11:50:01 494430301
>>494305
жду ваших бесценных ответов ,коллеги
Аноним 22/01/20 Срд 12:00:02 494433302
>>494429
> Сказать что-то хотел
>>494424
>тут никто не знает что такое магнитное поле, откуда оно берется и как работает. А если спросить, то тебя назовут шизиком
Аноним 22/01/20 Срд 12:17:23 494437303
>>494433
В треде ты спросить не пробовал, так откуда ты знаешь? По моим наблюдениям тут вахтер-шизик жидко пукает на каждую реплику, вопросов не задает, троллирует тупостью.
Аноним 22/01/20 Срд 12:23:51 494438304
>>494437
>В треде ты спросить не пробовал, так откуда ты знаешь?
Я спрашивал что такое магнитное поле, откуда оно берется и как работает в /sci/ 5 лет назад, тогда никто не смог ответить. Спрашивал 3 года назад тоже никто ничего не смог сказать. С тех пор /sci/ стал только раковее, так что ответа я точно не дождусь.
Аноним 22/01/20 Срд 12:50:42 494453305
>>494438
Пиздишь как дышишь. Отвечали на каждый из твоих вопросов, просто ты ответы понять не смог и начал кричать "маняматические фантазии", закрыв уши.
Аноним 22/01/20 Срд 12:51:38 494456306
>>494412
>Из магнетизма
Получается боженька так захотел?
Я не проваливаюсь к ядру, потому что жопой упёрт в твёрдый диван.
А как верхний магнит давит на нижний, но не сближается с ним? Они просто не хотят?
Аноним 22/01/20 Срд 12:52:28 494457307
>>494453
>Отвечали
Ну так покажи, что ты умный. Ответь что такое магнитное поле, откуда оно берется и как работает?
Аноним 22/01/20 Срд 13:00:51 494458308
>>494456
>потому что жопой упёрт в твёрдый диван
А жопа уперта благодаря какому фундаментальному взаимодействию, гений? Точно тому же, которое не дает жопе провалиться. Электромагнитному.
Аноним 22/01/20 Срд 13:01:26 494459309
>>494458
второй раз жопе = магниту
быстрофикс
Аноним 22/01/20 Срд 13:14:00 494460310
>>494458
Я хочу разобраться ещё немного подробнее, но ты, пидорас, только и делаешь, что ёрничаешь, да высираешь грубые встречные вопросы, как еврей. Спасибо за труд, иди нахуй.
Аноним 22/01/20 Срд 13:15:45 494461311
>>494457
> что такое магнитное поле
Название и есть ответ.
> откуда оно берется
Из магнита.
> как работает
Притягивает железки.
Свободен.
Аноним 22/01/20 Срд 13:17:17 494462312
>>494460
>Получается боженька так захотел?
>Я хочу разобраться
>высираешь грубые встречные вопросы
Найс. Это боженька тебя наказывает за ерничание, попробуй смиреннее.
Аноним 22/01/20 Срд 13:21:15 494463313
Аноним 22/01/20 Срд 13:32:36 494465314
>>494462
Ты посчитал, что фраза про боженьку - это нападки на состоятельность твоего ответа? Я не хотел грубить, я лишь спросил, есть ли объяснение более конкретное или магнетизм это свойство материи, которое существует как факт и никто не знает, почему это работает, как например само наличие заряда у фундаментальных частиц? Прости, если я сам начал эту перепалку, я пришёл в тред тупых вопросов не для этого. А теперь, спасибо за труд, иди нахуй, клоун.
Аноним 22/01/20 Срд 13:35:39 494466315
>>494465
> лишь спросил, есть ли объяснение более конкретное
Хочешь порвать пердаки знатокам /sci/ - спроси их о магнетизме и его природе.
Аноним 22/01/20 Срд 13:39:23 494467316
>>494466
Я здесь недавно, этот вопрос всегда был проблемным? И именно этот? Или срачи на любую тему не прекращаются даже по праздникам?
Аноним 22/01/20 Срд 13:39:41 494468317
>>494465
Пишу с хуя, все удивлены, что ты так рано ушел. Говорят твое место свободно, возвращайся. Твоя фраза про боженьку это не ерничанье, а корректное уточнение, магнетизм у магнита всего лишь следствие пространственного расположения замкнутого тока (магнитного момента) в частицах. Да, свойство, нет все знают класса с 6, что магнитное поле порождается движущимся зарядом. Как и то, что это магнитная составляющая ЭМ поля.
Аноним 22/01/20 Срд 13:43:45 494471318
>>494467
>Я здесь недавно, этот вопрос всегда был проблемным?
Да. Магнетизм это самая больная тут тема.
Аноним 22/01/20 Срд 13:45:55 494472319
>>494468
>магнитная составляющая ЭМ поля.
Что ты под этим понимаешь? Что такое "составляющая"?
Аноним 22/01/20 Срд 13:49:29 494473320
>>494472
Часть одной сущности, лол.
Аноним 22/01/20 Срд 13:52:08 494474321
>>494473
Ну как-то не очень, лучше как два отдельных поля это представлять, но сильно(прямо, полностью) влияющих друг на друга.
Аноним 22/01/20 Срд 13:52:37 494475322
>>494473
Звучит как циатат из библии.
Аноним 22/01/20 Срд 13:52:54 494476323
Аноним 22/01/20 Срд 13:53:54 494478324
>>494474
>лучше как два отдельных поля
Кому лучше-то? И как ты собрался преобразовывать координаты для них и объяснять почему они друг в друга переходят, а еще юзают одни фотоны?
Аноним 22/01/20 Срд 14:00:04 494484325
>>494478
>а еще юзают одни фотоны?
Так что, магнитное поле это фотоны?
Аноним 22/01/20 Срд 14:07:55 494486326
>>494484
Отдельно магнитное нет смысла обсуждать.
Квантовая электродинамика рассматривает ЭМ излучение, как поток фотонов.
Аноним 22/01/20 Срд 14:11:26 494488327
>>494486
>Отдельно магнитное нет смысла обсуждать.
Хорошо, а если рассмотреть его не отдельно, то что такое магнитное поле?
Аноним 22/01/20 Срд 14:34:16 494499328
>>494484
Нет, фотон это колеблющееся электрическое поле, которое колебает магнитное поле, которое колебает электрическое поле...
Аноним 22/01/20 Срд 14:34:50 494500329
>>494488
То что может двигать электроны и воздействовать на электрическое поле!
Аноним 22/01/20 Срд 14:37:02 494501330
>>494499
>которое колебает магнитное поле
А откуда берется это поле и что является квантом этого поля?
Аноним 22/01/20 Срд 14:37:19 494502331
>>494468
Вот электрон скастовал поле, потому что двигается.
Вот два магнита преодолевая силу трения и гравитации пиздуют друг к другу, если положить их близко на столе. Что их толкает? Что находится между двумя магнитами кроме бозонов?
Аноним 22/01/20 Срд 14:39:14 494504332
>>494488
Вообще по сути электричекого поля нет, всё электричество это поток электронов которые движет магнитное поле, но электроны имеют типа электрический заряд, который просто показывает с какой силой они воздействуют на магнитное поле при движении(ну и на самих себя).

В этом суть электрического поля. Т.е. его считай и нет, есть только магнитное, и то, как на него могут воздействовать частицы, а те которые на него могут воздействовать называют ЗАРЯЖЕННЫМИ(типа электрически заряженными, имеющими электрический заряд) ну и они ещё, эти электрически заряженные частицы, могут друг на друга воздейвовать, или через магнитное поле, или напрямую, притягиваясь или отталкиваясь.
Аноним 22/01/20 Срд 14:46:08 494512333
>>494504
Можно подробнее первую часть?
Аноним 22/01/20 Срд 14:58:51 494527334
Обожаю обсуждения магнитного поля :3
Аноним 22/01/20 Срд 15:04:15 494532335
>>494512
Электричество у тебя в розетке это поток электронов которые толкает магнитное поле в генераторе, а далее в проводах они толкают уже друг-друга, потому что заряжены одинаково.
Сила магнитного поля это и есть НАПРЯЖЕНИЕ, у тебя в проводах НАПРЯЖЕНИЕ в вольтах что есть сила магнитного поля в месте старта, в генераторе на электростанции и ТОК в амперах, это сколько электронов летит через сечение провода.

Но ещё электроны может пихать магнитное поле которое возникает в самом проводе при движении электронов по нему, тут хз что больше влияет.

И ещё электроны могут пихать друг друга без магнитного поля(но оно по любому появляется при их движении) например в химических источниках питания, батарейках и аккумуляторах, там просто в веществе происходит химическая реакция, появляется всё больше электронов и они начинаю друг-друга всё сильнее пихать, так появляется НАПРЯЖЕНИЕ, т.е. способность их пропихнутьсч куда-то, т.к. их много и они друг друга пихают сильно из-за большого их количества, например в провода пропихнуться и побежать по ним и Так получится ТОК.
Аноним 22/01/20 Срд 15:13:50 494542336
>>494532
>у тебя в проводах НАПРЯЖЕНИЕ в вольтах что есть сила магнитного поля в месте старта
Ебать как завернуто!
Аноним 22/01/20 Срд 15:14:41 494543337
>>494532
А вот если напряжение без магнитов добыть, то как так получилось?
Аноним 22/01/20 Срд 15:18:19 494547338
Аноним 22/01/20 Срд 15:39:04 494577339
>>494543
Ну я же говорю, идёт химическая реакция в которой из вещества батарейки или аккумулятора выходят электроны, их становится всё больше и больше, они заряжены одинаково, отрицательно, заряд минус у них, они друг от друга отталкиваются.
И вот они копятся и отталкиваются и это как вода накапливается в закрытом объёме и давление растёт.
Так же растёт и типа напряжение, напряжение это давление, т.е. напряжение это с какой силой электроны могут куда-то пропихнуться и с какой скоростью.
Например вода под большим давлением может пропихнуться через тонкий шланг быстро, а электроны под большим напряжением через тонкий провод быстро.
Аноним 22/01/20 Срд 15:41:42 494581340
>>494577
А если напряжение без магнитов и химических реакций, то что его рождает?
Аноним 22/01/20 Срд 15:49:18 494595341
>>494502
ПОЛЕ
В этом суть поля, поля это взаимодействия.

Ты бы мог воображать какие-то частицы, но из там нет. Это всё фантазия, абстракция, чисто для математических расчётов.
Их так и называют ВИРТУАЛЬНЫЕ, т.е. это костыль.
А некоторые шизики тупые насмотрятся научпопа и начинают всякую магию фантащировать.

Это как например с КВАНТОВОЙ НЕОПРЕДЕЛЁННСОТЬ. Например эти дауны думают что электрон в вакууме летит не по прямой, а например вихляя и делая такие развороты что оказывается в другой части вселенной, пока по замерам пролетит 1 метр по прямой.
Но нет блять, он летит строго по прямой если его так запустили, просто так физики пытались объяснить этим даунам что они не могут измерить его местонахождение не повлияв, поэтому не могут гарантировать что у него была именно траектория движения по прямой.
Но эти вернуские дауны конечно же начали всякую магию и шизотерию первым же делом воображать.
Но учёные ступили, это было же очевидно что так будет, с верунскими даунами всегда так.
Аноним 22/01/20 Срд 15:49:50 494597342
>>494595
>поля это взаимодействия.
А что такое взаимодействия?
Аноним 22/01/20 Срд 15:50:35 494600343
>>494501
Ты не сможешь доказать квантовость. Даже света.
Фотона нет.
Есть поле дробящееся бесконечно.
Аноним 22/01/20 Срд 15:50:52 494601344
Аноним 22/01/20 Срд 15:51:01 494602345
>>494600
>Есть поле дробящееся бесконечно.
Это как так? А как же кванты?
Аноним 22/01/20 Срд 15:51:44 494605346
>>494597
Изменение силы, направления силы или положения.
Аноним 22/01/20 Срд 15:52:07 494606347
dle2jhgn.jpg (19Кб, 600x455)
600x455
Аноним 22/01/20 Срд 15:52:36 494607348
>>494602
Чисто абстракция для удобства расчётов.
Аноним 22/01/20 Срд 15:53:09 494610349
>>494605
>Изменение силы
А что такое сила?
Аноним 22/01/20 Срд 15:53:57 494612350
>>494607
>Чисто абстракция
Квантов полей не существует? И ты можешь доказать это?
Аноним 22/01/20 Срд 15:54:19 494613351
>>494606
Ну ок, отрыв электронов, там тоже идёт просто накопление.
Они отрываются, физической силой(лол, если можно так сказать) а затем копятся как в первом и во втором случае, поэтому растёт напряжение.
Аноним 22/01/20 Срд 15:55:01 494615352
>>494613
>отрыв электронов
А что отрывает электроны?
Аноним 22/01/20 Срд 15:55:24 494616353
>>494612
Долбаёб верунский, когда до вас дойдёт что доказывается наличие, а не отсутствие.

>оряя!!1 докожите мне что моего бога нету!!1
Аноним 22/01/20 Срд 15:56:05 494618354
>>494610
А это то с какой интенсивностью что-то взаимодействует.
Аноним 22/01/20 Срд 15:56:34 494619355
>>494615
Тут хз, позже об этом подумаю на микроуровне.
Аноним 22/01/20 Срд 15:57:32 494623356
>>494616
>когда до вас дойдёт что доказывается наличие
Наличие квантов полей доказано. А вот сможешь ли ты доказать, что на самом деле все не так и квантов нет, а есть что там у тебя? Бесонечно дробящее поле?
Аноним 22/01/20 Срд 15:57:55 494625357
>>494618
>с какой интенсивностью
А что значить интенсивность?
Аноним 22/01/20 Срд 15:58:40 494626358
[Скорость борды: 100 п./час]
Вот что магнитное поле творит.
Аноним 22/01/20 Срд 15:59:25 494629359
>>494623
>Наличие квантов полей доказано.
Доказано чем?
Давай приводи доказательства, а я буду пытаться оспаривать. Не смогу-значит они есть.
Аноним 22/01/20 Срд 15:59:50 494631360
>>494625
Больше/меньше, сильнее/слабее.
Аноним 22/01/20 Срд 16:00:13 494633361
>>494629
>Доказано чем?
Экспериментами, чем же еще.
Аноним 22/01/20 Срд 16:00:18 494634362
>>494629
>Давай приводи доказательства, а я буду пытаться оспаривать. Не смогу-значит они есть.
*Это так делается.
Аноним 22/01/20 Срд 16:01:31 494639363
>>494633
>Экспериментами, чем же еще.
Конкретнее, пока ты только жидко пукнул показав насколько ноль в теме.
Аноним 22/01/20 Срд 16:03:27 494641364
>>494631
Т.е. если взять твое первое утверждение

>>494595
>поля это взаимодействия.

и для понимания того что такое взаимодействие использовать твое объяснение

>>494631
>Больше/меньше, сильнее/слабее.

То тогда все вокруг нас это поле и нет ничего кроме поля.
Аноним 22/01/20 Срд 16:05:57 494645365
>>494639
>Конкретнее
Ты правда не знаешь о фотоэффекте ? Жесть какая.
Аноним 22/01/20 Срд 16:09:02 494652366
>>494641
Он пропустил одно слово. Поля это переносчик взаимодействия.
Аноним 22/01/20 Срд 16:13:40 494661367
>>494577
Анон, пиздец какое тебе спасибо за объяснение напряжения! Серьёзно, я видел аналогии с током воды и силой электрического тока, благодаря которым всё понятно становится, а вот про напряжение - нет.
Аноним 22/01/20 Срд 16:17:40 494669368
>>494645
>Ты правда не знаешь о фотоэффекте ? Жесть какая.
Конкретнее, пока ты только жидко пукнул показав насколько ноль в теме.
Аноним 22/01/20 Срд 16:18:43 494672369
>>494641
>То тогда все вокруг нас это поле и нет ничего кроме поля.
Именно. А с чего ты решил что всё иначе?
Ты просто раньше об этом не задумывался.
Аноним 22/01/20 Срд 16:20:09 494675370
>>494661
Пожалуйста, я тоже долго охуевал когда закралась мысль что НАПРЯЖЕНИЕ это же блять просто пустое слово которое ничего не значит, и долго пытался понять что это такое.
А было это месяца 2 назад.
Аноним 22/01/20 Срд 17:59:40 494741371
Как происходит атрофия мозга на клеточном уровне и от чего она вообще зависит. Если Я ничем не примечательный, обычный мозг например в 20 лет решал йоба-уравнения на протяжении месяца нонстоп и позже не прикасался к ним вообще, но в 50 лет решил попробовать снова их решать, то:
1) Мне будет легче их заново учить если не брать во внимание снижение обучаемости с возрастом, как будто мне всё ещё 20 лет.
2) Я почти не забыл эти уравнения.
3) Я всё забыл и мне придётся их учить как в первый раз.
И, если есть, накидайте почитать про потерю навыков со временем.
Аноним 22/01/20 Срд 18:18:18 494750372
>>494390
>"Меньше поступает" неравно "больше вымывает".
Раствор бывает насыщенный и ненасыщенный. Чем он менее насыщен, тем более способен что-то ещё растворить. Не уверен, что это касается воды, но логика такая. Ну конечно, стоит только такой ненасыщенной воде попасть в рот, как она должна сразу стать самой обычной по мере загрязнения, меня просто конкретные исследования интересовали - так гадать можно сколько угодно.
Аноним 22/01/20 Срд 20:39:04 494780373
>>494750
Долго объяснять, но нет. Школьный курс неорганики + абзац в педивикии про жесткость + абзац про механизм впитывания и регулировки концентрации ионов кальция в организме. Все эти "гадать" исключительно из-за недостаточного уровня базовых знаний. Сомневаешься достаточно ли кальция - ешь орехи, пей молоко, заедай сыром (запивай дистиллированной водицей, только она невкусная).
Аноним 22/01/20 Срд 21:45:24 494787374
На фоне недавних событий подумал, а человечество вообще может умереть от эпидемии? Я думаю нет. На мой взгляд самые опасные вирусы - корь, эбола и ВИЧ. Их объединяет то, что они атакуют иммунные клетки, но корь и эболу организм может побороть. С ВИЧ человек живет десятки лет. Еще опасная испанка и подобные ей, провоцирующие аутоиммуную атаку, но ее тоже вроде можно перебороть. Так вот, как думаете, грозит ли человечеству апокалипсис от инфекции?
Аноним 22/01/20 Срд 22:13:24 494794375
ras.mp4 (4730Кб, 1280x720, 00:00:34)
1280x720
Вспомнил песню смалтовн бой в рамках обдумывания одной идейки у себя в голове, а спустя 3 секунды натыкаюсь на неё в видео на ютубе. Случается, что вспомнишь говно, а вот и оно. Еще очень часто люди говорят о том, что пришло мне в голову секунд 10 назад. Хули матрица так палится передо мной?
Аноним 22/01/20 Срд 22:20:28 494800376

>>494787
Зависит от определения апокалипсиса.
Аноним 22/01/20 Срд 22:39:53 494802377
>>494794
+
Жиза, постоянно.
Аноним 22/01/20 Срд 23:03:05 494803378
>>494794
Когнитивные искажения. Это несовершенство твоего мозга, по факту, палится, не матрица.
Аноним 22/01/20 Срд 23:12:53 494804379
>>494803
Схуяли? Где и какая логика в том что ты сказал?
Аноним 22/01/20 Срд 23:13:32 494805380
>>494787
>Я думаю нет.
Грозит. Если выбрать удачные сочетания смертельности, заразности и инкубационного периода - пизда нашему укладу жизни. Почти как в Plague Inc. Там останутся ли дикари из процов вытачивать наконечники, или в убежищах 300 человек выживет - не так уж важно.
>самые опасные вирусы - корь, эбола и ВИЧ.
Хуйня уровня детсада. Какие вариации из оспы и вируса ракоспида наделали в лабораториях за эти годы никто не расскажет. Не исключено, что в одной из пробирок уже есть агент, способный уничтожить современную западную цивилизацию. Европка вон на 90% вымирала, а представь умрут все инженеры, водители, технологи, остановятся электростанции, селхоз, транспорт, закроют дороги и порты... Это же коллапс сразу, может даже города вымрут от голода быстрее, чем от эпидемии.
Аноним 22/01/20 Срд 23:14:56 494807381
>>494794
Был период, когда матрица палилась передо мной так, что я подумал, что схожу с ума.
Может это несовершенство алгоритмов?
Аноним 22/01/20 Срд 23:16:56 494808382
Cognitivebiases[...].png (522Кб, 1280x905)
1280x905
>>494804
С того ли, заебали шизики, которые приходят спросить сколько 2х2, да определение слова "жопа", а даешь кейворды, они начинают спрашивать, что такое умножения и ягодицы. Хоть понимаете, что "тупых вопросов", а не "ленивых уебков" тред?
Аноним 22/01/20 Срд 23:23:26 494809383
cognit.jpg (1135Кб, 3508x2481)
3508x2481
Аноним 23/01/20 Чтв 00:05:15 494812384
>>494808
Ты наверное как раз жопой и думаешь. При чём тут механизмы мозга работы с информацией, если события происходят независимо от тебя. Какая разница, что ты можешь вспомнить, заместить, создать ложное воспоминание, упростить и т.д., если в случае с "матрицей" два простых события становятся очень маловероятными, если происходят подряд.
Многим известен эффект, когда ранее неизвестные тебе афоризмы, определения, значения слов начинают всплывать повсеместно после того как ты о них узнаёшь. Самое пресное и лёгкое объяснение - теперь ты подмечаешь всё это, узнав о смысле и приняв новую информацию как знакомую, но я использую русский язык всю свою жизнь и легко могу подметить слово отсутствовавшее в моём поле информации несколько десятков лет, а после первой встречи оно "обкатывается" множество раз за короткое время, будто информация навёрстывает пропуски. Со знакомым уже несколько лет подмечаем такие события и делимся ими время от времени. Буквально недавно вообще впервые в жизни узнал, что яйцо сваренное вкрутую без скорлупы называется пашот. В жизни не ел, не слышал, не видел, никто в семье блюда из яиц не жалует. Услышав странное слово на улице, тут же загуглил. Три дня подряд я не мог избавиться от этой хуйни, услышал несколько раз в кафе, по телевизору, наткнулся в интернете и один раз услышал от матери. Через несколько дней я перестал слышать и видеть яйцо пашот от слова совсем. И так постоянно. Несмоненно, мозг начал обращать внимание на это слово, но я и не пропускал его мимо ушей десятилетия до этого, я его просто не знал. Какая вероятность плотности этих событий? При чём тут когнитивные искажения? Идиотская отговорка, учитывая, что о таких совпадениях время от времени докладывает каждый второй человек.
Аноним 23/01/20 Чтв 00:15:16 494814385
>>4948в
Аноним 23/01/20 Чтв 00:18:51 494815386
>>494812
О, охуенчик, даун не только обиженка, он ещё и объективную истину в своём личном восприятии откопал. Поздравляю, ты только что гранату. Скоро до уровня познания античного философа преисполнтшься (вряд ли).
Аноним 23/01/20 Чтв 00:48:49 494821387
>>494787
>Так вот, как думаете, грозит ли человечеству апокалипсис от инфекции?
Он уже происходит. Каждый год от обычного гриппа и вызванных им осложнений умирают ок. 650 000 человек и еще несколько миллионов переносят его в очень тяжелой форме приводящей к сокращению продолжительности жизни.
Аноним 23/01/20 Чтв 02:00:42 494828388
>>494780
>Все эти "гадать" исключительно из-за недостаточного уровня базовых знаний.
Да, разумеется, потому я и спрашиваю.
Спасибо. Могу ещё придраться, что речь идёт вовсе не только про кальций, но не буду.
Аноним 23/01/20 Чтв 03:12:35 494835389
>>494828
Ну так не стоит, там почти все кальций и магний, кальция больше (порядка 4/1 - 10/1, судя по табличка из поиска), остальное можно не считать. А в случае дестиллята - наоборот, отсутствие кальция и немного магния.
Аноним 23/01/20 Чтв 03:14:42 494836390
>>494835
Дистиллята, офкос
скоропочин
Аноним 23/01/20 Чтв 05:19:47 494851391
>>494794
Ищи про синхронии, если ещё не слышал про таких. (Не, едва ли там будет их объяснение.)
Аноним 23/01/20 Чтв 05:33:26 494856392
>>494851
Ты Юнга читал сам? Совесть не мучает этого оккультиста в псевдонаучную тематику тащить? Кто дальше в треде будет объяснять по хардкору за науку, Нострадамус, Кроули, Кастанеда?
Аноним 23/01/20 Чтв 06:12:08 494859393
>>494856
Не читал, просто разместил объяву. Два вопроса:
1. Какие конкретно претензии к означенным авторам?
2. Можешь предложить что-то лучше?
Аноним 23/01/20 Чтв 06:31:44 494862394
>>494859
>Какие конкретно претензии к означенным авторам?
Тематика не та, в /mg/ с ними. И вообще, Юнга не читал (ранний еще вполне), этих всех прочел? Или только слышал в соседнем треде?
>Можешь предложить что-то лучше?
Уже. Комбинируем когнитивные искажения и получаем нужную картину. Это как с дежа вю, тупой верун объясняет через прошлые жизни, наукобоярин - через искажения восприятия, например ошибку записи кратковременной памяти в долговременную. Научный метод, бритва там, все дела.
Аноним 23/01/20 Чтв 06:43:20 494863395
>>494862
>этих всех прочел?
Нет, этих тоже не читал.
>Комбинируем когнитивные искажения и получаем нужную картину.
>наукобоярин - через искажения восприятия
А толку?
Аноним 23/01/20 Чтв 06:55:27 494864396
>>494411
Но я могу оценивать глубину по параллаксу при движении камеры, сопоставляя перекрытия одного объекта другим, зная размеры типичных объектов, по снижению контраста и посинению удаленных объектов
Аноним 23/01/20 Чтв 07:04:55 494865397
>>494864
Ты увидел глубину на картинке по параллаксу? Заметь, без типичных размеров, предварительного обучения, нужного освещения, там просто есть 3д картинка. Для всех, кроме одноглазых, ну может еще дальтоников и косоглазых.
Аноним 23/01/20 Чтв 07:39:40 494869398
image.jpeg (142Кб, 400x388)
400x388
Аноним 23/01/20 Чтв 07:53:01 494870399
DiZlJMhX4AEfUh.jpg (35Кб, 500x467)
500x467
>>494869
C U T T I N G E D G E
U
T
T
I
N
G

E
D
G
E
Аноним 23/01/20 Чтв 07:57:53 494872400
Аноним 23/01/20 Чтв 08:11:20 494873401
Аноним 23/01/20 Чтв 08:16:41 494874402
>>494873
>Оценивает оптическую иллюзию на плоском экране.
>Доказывает этим второстепенность стереозрения в оценке расстояния.
Шизик, ты?
Аноним 23/01/20 Чтв 10:03:36 494899403
>>494488
Поле это какая-то математическая хуйня для описания каких-то взаимодействий. Магнитное поле - для взаимодействия движущихся зярядов.
Аноним 23/01/20 Чтв 10:15:15 494901404
Аноним 23/01/20 Чтв 11:02:44 494907405
>>494899
>Магнитное поле
Что это?
Аноним 23/01/20 Чтв 11:45:57 494911406
>>494907
Хуита в пространстве, которая заставляет поворачивать движущиеся заряды.
Аноним 23/01/20 Чтв 12:53:44 494924407
>>494911
>Хуита в пространстве
Лол, отличная формулировка.
Аноним 23/01/20 Чтв 13:09:52 494925408
Аноним 23/01/20 Чтв 13:25:22 494927409
Аноним 23/01/20 Чтв 13:30:05 494928410
>>494927
Все математика, даже небо, даже твоя мамка.
Аноним 23/01/20 Чтв 13:39:38 494930411
Здравствуйте. Можно ли полноценно одновременно слушать аудиокнигу, и читать текст, иного содержания, нежели книга, и воспринимать инфу, как если бы ты делал это по отдельности?
Аноним 23/01/20 Чтв 13:51:34 494932412
>>494930
И еще вопрос.
Обязательно ли читать сознательно, то есть концентрировать внимание именно на чтении, на каждом слове, смысле?
Каждый наверняка отвлекался от чтения на всякие раздумья, и забывать, как прочел х страниц. Качество усвоения такой инфы сильно отличается от осознанно прочитанного?
Я просто заметил, что не всегда осознанно прочитанное усваивается на 100, и забывается конечно, в свою очередь, инфа, которую ты получил, не концентрируясь на ней, может всплыть в памяти.
Справедливо ли различать мозг и сознание, как нечто целое и его часть? Почему людям не очевидно, что сознание это не весь мозг? Точнее сознание, это результат деятельности мозга, как видеосигнал, идущий на монитор, результат работы компьютера. Может я ошибаюсь, и это не очевидно, или ложно?
У меня есть предрасположенность к шизофрении?
Почему, если выражаться прямым текстом, без вранья и увиливаний, и не принимать ложь других людей, тебя считают дебилом, шизиком, человеком без души?
Аноним 23/01/20 Чтв 13:53:44 494933413
ggggggggggggggg[...].png (774Кб, 890x792)
890x792
>>494930
>>494932
Кому не нравятся слова "осознанно", "сознание", "подсознательно", предложите альтернативу, я не знаю, как еще написать, специально взял в кавычки, что бы вы не доебывались.
Аноним 23/01/20 Чтв 14:27:10 494935414
img-11.jpg (59Кб, 800x420)
800x420
>>494932
>Справедливо ли различать мозг и сознание, как нечто целое и его часть?
Справедливо ли различать комп и винду? Да.
>Почему людям не очевидно
Потому как люди себе любят льстить, например добавили после Homo слово Sapiens.
>результат деятельности мозга
Даже хуже, это только окно программы, не имеет смысла без остальной части софта или железа, на котором крутится. Ну и банальное - это видимое окно программы относительно слабо связано с процессами, которые идут внутри.
>У меня есть предрасположенность к шизофрении?
Если у родственников была/есть, то риск выше.
>если выражаться прямым текстом, без вранья и увиливаний
Потому как общение/психология работает не так, по сути это патология общения, на уровне Туретта.
От банального "здравствуйте" не равно пожеланию здоровья, до семантики и философии (прямой текст не равен правде, правда субъективное философское понятие, у каждого в голове своя дефиниция слов, семантический шум и так далее). Кратчайший путь донести мысль в коммуникации - как в неэвклидовой геометрии - чаще всего не прямая между А и Б.
Аноним 23/01/20 Чтв 14:35:51 494939415
>>494928
Наверно не вся. Что-то типа этого.
ru.wikipedia.org/wiki/Тензор_электромагнитного_поля
Аноним 23/01/20 Чтв 14:36:54 494940416
>>494935
>Sapiens
Тоже проигрываю с этого, еще слово "воля" мне очень доставляет.
Аноним 23/01/20 Чтв 14:47:52 494942417
552.png (45Кб, 199x103)
199x103
Аноним 23/01/20 Чтв 15:27:25 494946418
>>494942
Ты меня прям сделал веселиться.
Аноним 23/01/20 Чтв 17:45:05 494973419
Можно ли назвать глупым сознательное существо, которое:
- обладает хорошей памятью, сообразительностью, смекалистостью
- может без запинок применять на практике приобретенные умения
- НЕ может "созидать" и придумывать что-либо разительно новое
?

На примере создания музыки: в состоянии синтезировать мелодии уже написанных композиций, но не может выдумать свою.
На примере студентоты: может написать работу на основе ранее обдуманных работ других авторов, просто переставив слова по-своему, но не может придти к самостоятельным выводам и написать работу из головы.

>>494930
В 15 лет я мог играть в мафию 1, слушать фильм и одновременно обдумывать прочитанную книгу, а сейчас немного трудно просто вспоминать что-то и воспринимать информацию, потому что первое занимает всё сознание и глушит остальное. Рассеянных людей либо очень сильно захватывает одна мысль, либо они обладают очень маленькой сферой внимания, ну, либо думают о 10 вещах разом.
Аноним 23/01/20 Чтв 17:50:24 494976420
>>494973
>глупым
На харкаче можно назвать дауном любого анона, будь он хоть кандидатом, хоть профессором. А вообще это абстракция, все еще можно называть кого и как хочешь, критерия глупости нет.
>выдумать свою
Еще поди и всего семь сраных нот пользует, как умственно отсталый?
Аноним 23/01/20 Чтв 19:10:10 495006421
>>494973
Написание музыки, как и любое креэйторство - складывание уже имеющихся пазлов, в различных комбинациях. Любой композитор слушал ранее множество схожих мелодий, вопрос лишь на сколько % он их передрал. Изолированный, гипотетический, с детства, на необитаемом острове человек, не сможет петь, говорить, объяснять, он будет разумен и креативен, насколько может быть собачка или обезьянка.
Ньютон не придумывал ничего нового, он лишь сделал выводы, из уже имеющихся фактов. Вообще возможно, что значимые умозаключения и произведения искусства это простой рандом, пазл получился полезный, или приятный, в множестве подобных, менее значимых, вплоть до полной какофонии и безумия. Тем не менее те редкие, рандомные ньютоны и вивальди, позволяют стаду считать себя разумными и творческими. К сожалению мы "плесень", и ничего больше.
Аноним 23/01/20 Чтв 19:29:41 495010422
>>495006
>Ньютон не придумывал ничего нового
Основы диффура, дифференциальных исчислений;
основы интегрального исчисления;
построил телескоп-дефлектор;
разложил белый свет;
описал систему мира (способ посчитать положение тел, если известны силы и начальные условия);
изобрел ряд тейлора (хоть и не довел до конца);
еще там пару открытий в аэро/гидродинамике.
>ничего нового
>ничего
Аноним 23/01/20 Чтв 19:51:28 495018423
>>495010
>изобрел новый метод окучивания картошки
Всё так, модернизировал телескоп, пересмотрел методы расчетов, что не так? В отрыве от предыдущих исследователей, его результат был бы 0. 100% результат его работы, отталкиваются на исследования предшественников, либо на свои собственные, которые в свою очередь...
Ну очевидно это всё демагогия. Человек без культуры ничто.
Аноним 23/01/20 Чтв 20:01:28 495020424
>>494925
Т.е. в теме ты не разбираешься а только посмотрел видева на ютубе?
Аноним 23/01/20 Чтв 21:07:12 495027425
Анончики, подскажите годные англоязычные каналы на Ютубе вроде нашего Архэ и Постнауки. Тоесть с популяризацией, но не для совсем даунов.
Аноним 23/01/20 Чтв 21:19:12 495032426
>>495018
У тебя важный и правильный посыл про одинаковые кирпичики креативности, а потом сразу БАБАМ, из кирпичей дома не строят, всё просто свалили с самосвала удачно, цемент мимо летел - прилип, чудом прокопали подъезд, так и живем.
Аноним 23/01/20 Чтв 21:33:27 495033427
>>495027
>с популяризацией, но не для совсем даунов
/0
Аноним 23/01/20 Чтв 21:39:25 495034428
Какие есть научные обоснования для возможности посмертного существования?
Аноним 23/01/20 Чтв 21:40:22 495035429
>>495033
Вот кстати да, очень не хватает научпопа, но хоть немного с формулами и примерами расчётов.

Т.е. вот все научпоперы сняли видос о какой-то теме, начальный, и дальше нихера на углубление в тему не двигаются, а тупо повторяют опять и опять поверзностные знания по теме, одно и то же.
Аноним 23/01/20 Чтв 21:40:40 495036430
>>495034
Никаких. Организм умирает, цнс прекращает работать, существование души доказать никто не смог.
Аноним 23/01/20 Чтв 21:49:08 495037431
>>494787
Вроде уже пояснили что ебола опасна лишь для африканцев, у которых нет доступа к чистой воде и медикаментов. Вич миф. Корь какая-то детская болезнь
Аноним 23/01/20 Чтв 21:55:27 495038432
Аноним 23/01/20 Чтв 21:55:57 495039433
>>494932
> человеком без души?
Ты рыжий?
Аноним 23/01/20 Чтв 21:59:47 495040434
>>495035
>не хватает научпопа, но хоть немного с формулами и примерами расчётов
Все первые главы учебников для первых курсов именно то о чём ты говоришь.
Аноним 23/01/20 Чтв 22:02:10 495041435
>>495040
Нихуя, я хочу по конкретной теме, а не общую физику.
Аноним 23/01/20 Чтв 22:04:12 495042436
>>495041
Первые главы по конкретной теме ¯\_(ツ)_/¯
Аноним 23/01/20 Чтв 22:07:26 495043437
>>495036
а полностью на 100% опровергнуть?
Аноним 23/01/20 Чтв 22:14:58 495044438
Аноним 23/01/20 Чтв 22:15:23 495045439
>>495037
>Вич миф
Лол. А под микроскопом что видно?
Аноним 23/01/20 Чтв 22:16:08 495046440
>>495045
Микроскоп это миф
Аноним 23/01/20 Чтв 22:18:34 495047441
>>495046
а я видел однажды микроскоп своими глазами, по крайней мере мне сказали что это он
Аноним 23/01/20 Чтв 22:21:41 495049442
Аноним 23/01/20 Чтв 22:39:32 495050443
>>495043
Как только опровергнешь чайник Рассела - поговорим.
Аноним 23/01/20 Чтв 22:58:27 495051444
>>495032
Просто рассуждаю, да и ты наверное не совсем меня понял.
Аноним 23/01/20 Чтв 23:15:27 495052445
Аноним 24/01/20 Птн 01:07:39 495068446
15798134814970s.jpg (4Кб, 200x133)
200x133
Есть версии вымирания динозавров от вирусов?
Аноним 24/01/20 Птн 05:22:07 495117447
Аноним 24/01/20 Птн 06:35:51 495126448
>>495125
Ты просто в своем познании лишку преисполнился, жди когда попустит, еще и простыню высрал, где намешано в кучу все. Еще и рассуждения типа "если в магме сделать человечков, в точности как людей, то можно сделать твою мамку и ее ебать, это называется детерминизм".
Аноним 24/01/20 Птн 06:51:29 495128449
>>495126
Я по делу писал, а ты всё обосрал и перековеркал.
>Еще и рассуждения типа "если в магме сделать человечков, в точности как людей
Я вообще не так написал.
Я написал следующее:
>Это значит, что если где-нибудь, на какой-либо планете, солнечном пятне, или потоках магмы или ещё где,
>возникнет информационная система идентичная информационной системе
>бла-бла-бла...
Откуда тебе знать, что в различных средах не могут возникнуть носители информации,
которые в состоянии будут, в совокупности своей, нести информационные процессы и информационне системы - более сложные, нежели обычные сигналы?

Например, вот тут - чипы, способные работать на Венере:
https://telegraf.com.ua/tehnologii/3157994-v-nasa-sozdali-chip-sposobnyiy-rabotat-na-poverhnosti-veneryi.html
Там около 400 цельсия, вблизи поверхности!
Из них уже можно делать кластеры, суперкомпьютеры всякие, и так далее.

Ну так вот, а где гарантия, того, что в природе, в результате самоорганизации материи,
не могут возникнуть подобные системы в средах более агрессивных?
Например, подобные системы могут быть реализованы в виде квантовых вентилей,
из каких-нибудь нейтрино, в ядре Солнца или на его поверхности (под сильными магнитными полями),
а нести информацию они могут, в совокупности своей - через различные квантовые эффекты,
квантовую запутанность, квантовую телепортацию и обмен инфой между собой,
сохраняя при этом четко-определённую как структуру носитлей инфы, так и структуру носимой ими - информационной системы.
Аноним 24/01/20 Птн 07:39:43 495134450
>>495128
Осознаю сизифов труд. Ты должен понять тонкую грань, между научной абстракцией и шизофазией, например пока ты говоришь: слоны высокоинтеллектуальные социальные животные, мухи обладают сверхбыстрой реакцией, пчелиный рой коллективно работает сложнее, чем особи отдельно (самоорганизуется) - все хорошо.
А вот когда ты продолжаешь: Так что ничего нам не мешает создать слонопчеломух, которые будут во всем лучше людей и, вероятно, за неделю смогут построить город на марсе - вот тут уже, хьюстон, у нас проблемы.
Да, в рамках науки можно потеоретизировать о вариантах неуглеродной, и даже некремниевой жизни, но без душ, точных копий и информационных систем, как минимум пока ты не дашь точных, научнонепротиворечивых определений этих твоих систем, души, и прочего. А уж мысль, что на другом принципе жизнь будет "точно как мы", вообще пропахла говном верунством и антропоцентризмом настолько, что аж из штанины кал сыпется при ходьбе.
Аноним 24/01/20 Птн 08:22:29 495138451
>>495134
>Осознаю сизифов труд.
От чего же? Просто увидел пост, и написал что думаю. Свободное общение же.
>Ты должен понять тонкую грань, между научной абстракцией и шизофазией

>например пока ты говоришь:
>слоны высокоинтеллектуальные социальные животные,
>мухи обладают сверхбыстрой реакцией,
>пчелиный рой коллективно работает сложнее,
>чем особи отдельно (самоорганизуется) - все хорошо.
>А вот когда ты продолжаешь:
>Так что ничего нам не мешает создать слонопчеломух,
>которые будут во всем лучше людей и, вероятно, за неделю смогут построить город на марсе - вот тут уже,
>хьюстон, у нас проблемы.
А на другом конце провода сидят невъебеннейшие учёные...
И они такие... Оп-па! А так можно? Так-так... Сейчас проверим...
Ага, слоно... пчело... Мухи! Да, есть такая сверхсложная модель организма, ебать!
Даже ДНК стыкуется, аллели генов имеются, всё просчитанно, осталось лишь материализовать.
Подобные виды животных уже жили в каком-нибудь "архее", но ДНК из останков не было получено из-за их древности,
оно было только вчера выведено в результате работы проектов добровольных вычислений...
Ну ты понял...
Для науки это может быть сигналом и путём развития, и направлением для дальнейших исследований,
для чего и существует свобода слова, и свободное общение на двощах.

Для тебя же - то что является хоть и неполной, но научной абстракцией,
будет просто бредовой шизофазией, так как ты просто не видишь взаимосвязей,
и из-за большого количества букофф в "ненавитстной простыне необмоченной",
и вероятно - просто не хочешь их увидеть, а захочешь позвать мочератора.

Тем не менее, по мере того, как ты всё-же начинаешь видеть эти взаимосвязи,
ты начинаешь понимать нечто большее, чем та исковерканная хуета,
нарисовавшаяся в твоём воображении, впервые.

>Да, в рамках науки можно потеоретизировать о вариантах неуглеродной, и даже некремниевой жизни,
>но без душ, точных копий и информационных систем,
>как минимум пока ты не дашь точных, научнонепротиворечивых определений этих твоих систем, души, и прочего.
По крайней мере, я это определение, пусть и не точно, пусть и не полно, но всё-же ввёл,
в качестве вводных данных, к своему посту,
в отличие от анона с поста которого начался этот диалог.
Можете дополнить это определение или модифицировать.
>А уж мысль, что на другом принципе жизнь будет "точно как мы",
>вообще пропахла говном, верунством, и антропоцентризмом настолько,
>что аж из штанины кал сыпется при ходьбе.
Ну почему же?
Тот же игрок в какой-нибудь кампании игры Battlefield 5: https://www.youtube.com/playlist?list=PLkJoTOgcypA80o_3nfmHw8Tj8LOlO9dfj
"ощущает" себя вполне себе игроком, видит свои руки, автомат, и цели,
причём независимо от того, на каком компе, с каким процессором, с какой видеокартой, и каким жестким диском запущена эта игра,
при этом, при однозначно определённом управлени, этот игрок всё так же проходит игру к финишу,
не отвлекаясь особо на то, ЧЕМ ЖЕ НА САМОМ ДЕЛЕ ОН ЯВЛЯЕТСЯ.
Аноним 24/01/20 Птн 08:59:26 495142452
t-8q0WOL6eY.jpg (7Кб, 200x200)
200x200
>>495138
>невъебеннейшие учёные
>ДНК стыкуется
>Подобные виды животных уже жили в каком-нибудь "архее"
>Для науки это может быть сигналом и путём развития
>ты просто не видишь взаимосвязей
Пикрил
>чем та исковерканная хуета
Шизики все однотипны, считают, что если им показалось, что исковерканная хуета - это умная мысль, то осталось только убедить весь мир в том же, вместо принятия факта, что иногда банан - это просто банан.
>свобода слова, и свободное общение на двощах
Ты превратно ее понимаешь - часть свободы слова это самоорганизация и самоограничение, в случае невыполнения этих пунктов реципиенты информации вынуждены контролировать и ограничивать эти швободки, дабы не превращать общение в анархию и свалку. Чтобы тебе было понятно - свобода слова это когда ты можешь высказать свои идеи и убеждения и нет общего запрета на подобное высказывание, но если твой посыл контрконструктивен и контпродуктивен, тебя должны потереть, а потом забанить, т.к. ты сам не остановишься. А вот г-н Эйнштейнов-Лапласов, разумно и аргументированно высказывающий в чем-то сходные мысли так, что они могут, в теории, принести пользу - наоборот, получит разворот в читаемом издании.
>По крайней мере, я это определение, пусть и не точно, пусть и не полно, но всё-же ввёл,
Так нет же, не можешь в определения (а ты не можешь), так пиши четкие критерии, что будет считаться тем-то тем-то. И српазу вылезут все косяки, например для сходного восприятия неуглеродной жизнью, этой жизни понадобятся глаза и уши, что довольно абсурдно для существ из ядра звезды, нет никаких предпосылок, почему им нужны будут органы восприятия видимого света в среде, где это не дает ничего, и так далее.
>Можете дополнить это определение или модифицировать.
How about no?
>на каком компе
>с каким процессором
x86(x86-64)
>с какой видеокартой
Поддерживающей стандартные API и команды.
>каким жестким диском
API, файловая система из стандартных.
Никогда такого не было, и вот ты опять обосрался. Ты хоть в одной области что-то смыслишь, дурик? Короче компьютерные аналогии даже не трогай руками, только выдаешь насколько ты безграмотный. Ты можешь запустить эти вот игры под виндой (и гораздо реже под линухом) только потому, что умные дядьки инженеры и кодеры сделали такой уровень абстракций, чтобы эта совместимость вообще была. Причем все процы во всех этих писюках собраны на одной основе, одними методами и поддерживают строго определенные наборы инструкций. Ну так, для общего развития тебе.

Аноним 24/01/20 Птн 10:07:57 495151453
>>492883 (OP)
ЛАДНО, АНОНЫ, БЫЛО ПРИЯТНО, НО Я ВАЛЮ ИЗ /SCI/ НА НАНОБОРДУ
А ТО ТУТ МОЧЁЙ ВОНЯЕТ
КАКИЕ-ТО УЁБКИ - ТУПО ТРУТ ПОСТЫ, И ДАЖЕ ЗЛОУПОТРЕБЛЯЮТ БАНАМИ
С РАЗМЫТОЙ ПРИЧИНОЙ ШИТПОСТИНГ, ЧЁТКИМ КРИТЕРИЕМ НЕ ЯВЛЯЮЩЕЙСЯ.
Аноним 24/01/20 Птн 10:26:38 495160454
Аноним 24/01/20 Птн 11:47:11 495171455
>>495117
Пруфы или мастодонт
Аноним 24/01/20 Птн 11:49:43 495172456
>>495171
Беру мастодонта.
Аноним 24/01/20 Птн 15:47:42 495206457
>>495151
>постит шизу
>шизу трут
По мне так работает как надо. Ещё бы дебичей с магнитным полем потёр, вообще заебись бы было.

>>495044
Какие именно темы интересуют?
Аноним 24/01/20 Птн 16:32:16 495218458
>>495206
>Какие именно темы интересуют?
Очевидно, что никакие. Тут, заметь, вообще не очень-то любят читать, а кто любит - не будет спрашивать, прост скачает и почитает.
Лучше скажи, видел ли псевдоучебники, чтобы с мутью похлеще рентв, там для раен каких-нибудь, поугарать чисто? Ну или там кандидатскую-докторскую на тему плоской земли, ну ты понел.
Аноним 24/01/20 Птн 16:44:51 495219459
>>495206
>Ещё бы дебичей с магнитным полем потёр,
А как ты понял, что они дебичи? Ты знаешь, что такое магнитное поле, откуда оно берется и как работает?
Аноним 24/01/20 Птн 17:58:10 495229460
>>495219
Тебе уже отвечали, силовое поле, из движения зарядов/магнитного момента, ну или электрическое поле в него переходит (а точнее это одна штука - ЭМ поле). Работает оказывая воздействие на все те же движущиеся заряды или магнитные моменты (объекты с ними). Раз обожаешь обсуждения магнитного поля, сам бы что-нибудь написал уже по теме, целый цирк ты наш.
Аноним 24/01/20 Птн 18:22:53 495233461
Когда я топлю печку с открытым поддувалом, иногда дым начинает просачиваться через щели наверху печки и прекращает, если закрыть поддувало. Причем иногда все нормально и с открытым поддувалом. Умные люди, поясните, почему так.
Аноним 24/01/20 Птн 18:23:47 495234462
>>495206
>Какие именно темы интересуют?
Ой, это надо пересматривать научпоп.

Вроде про попова, когда его видео смотрю, думаю бля, видео новые, а он тупо опять и опять пиздит одну и ту же херню, ну как лектор совковый, 40 лет читает лекции со своей тетрадки но у же по памяти которые записал ещё в совке когда студентом был.

На одной части он конечно не виноват, на тех видео где он просто ЛЕКЦИИ читает, и его снимают и выклдаывают это, т.к. тут очевидно что повторяться будет.
Но вот видео которые он записывает сам, в виде типа какого-то КУРСА, то там бы надо бы уже и углубляться понемногу в тему, с формулками, модельками, схемами расчётными приближёнными хотя бы простенькими и сильно приближёнными на доске...


А что конкретно интересует-из сразу вспомнившегося, так это как вообще в принципе потенциально может образоваться именно чёрная дыра, т.е. как материя может провалиться за горизонт событий, если для стороннего наблюдателя, т.е. для нас, время процессов возле горизонта бесконечно замедляется. Т.е. теоретически, согласно эйнштейновской модели (не уверен, по логике), которая полностью и везде сейчас подтверждается, образование чд невозможно.
Хотелось бы какую-то мат-модельку, показывающую что горизонт событий таки может догнать вещество, чтобы оно оказалось за ним, а "находящееся бесконечно близко к нему через бесконечное время, образуя объект бесконечно похожий на чёрную дыру"(но ей не являющийся, лол.)

Второе-про взрывы суперновых. У них есть направленнсть как пишут, т.е. два луча на полюсах вращения. Во первых почему так происходит, объяснение с точки зрения физики. А второе-это больше информации по этому явлению, нигде не нашёл. Т.е. если это лучи, но конусы, очевидно, а не классические лучи, то какой у них угол расхождения, очевидно что у них интенсивность излучения будет ослабевать меньше чем обратноквадратично. Плюс больше про мощность взрывов сверхновых, сколько в лучах энергии выделится, сколько во все стороны равномерно, сколько дойдёт до потенциально находящейся планеты на пути этого на разном расстоянии и эквивалент этого.

Ещё почему теоретически считается что сингулярность внутри вращающейся чёрной дыры представляет собой тороид, а не точку. Подробно

Ещё интересный вопрос про чд и чёрные дыры задали тут когда тоже писал про падение в чд и невозможность этого, из-за за земедления времени для стороннего наблюдателя.
Мне сказали что замедляются процессы возле горизонта, но сказали что ДВИГАТЬСЯ в космосе/пространстве ты будешь так же.
Я с одной стороны понимаю что все процессы во вселенной это есть движение элементарных частиц через пространство, и если процессы замедляется, то и падение/движение возле горизонта замедлится для стороннего наблюдателя, но хотелось бы подробне. Почему?
Потому что пространство растянуто?

И вытекающий из этого вопрос смежный про выпускание фотона в обычном и растянутом пространстве.

Это пока что сходу вспомнил
Аноним 24/01/20 Птн 18:35:22 495236463
Ну охуеть, высер веруна "докажите мне что моего бога НЕТ" про чайник рассела не потёрли, а те посты где его головой в говно макали потёрли, заебись.
Аноним 24/01/20 Птн 20:49:45 495250464
Screenshot20200[...].jpg (289Кб, 1080x1325)
1080x1325
Объясните мне эту хуйню, плес, с осцилляторами у Фейнмана. Почему получается спираль, я не понимаю, и как это объясняет например правило наименьшего времени для света, проходящего сквозь среду?
Аноним 24/01/20 Птн 21:02:54 495253465
maxresdefault.jpg (142Кб, 1280x720)
1280x720
Реально ли приспособить пикрелейтед на автомобиль? Без мегаразмеров ради увеличения мощности, амеры чет там в 70х лепили, какая то хуйня получилась на 70 л.с.(или соврали?). В любом случае это дело похерили, разве что для генератора или насоса его используют.
Из того что я понял:
Сделать силовую установку компактнее можно заменив рабочий газ(воздух) на гелий, и накачав в цилиндры его до 100 атмосфер.
Аноним 24/01/20 Птн 21:08:14 495254466
>>495253
>Сделать силовую установку компактнее можно заменив рабочий газ(воздух) на гелий, и накачав в цилиндры его до 100 атмосфер.
Ты это понял или подсмотрел?
Потому что такие есть, и твой вопрос в таком случае бред, просто бери и ставь.
Аноним 24/01/20 Птн 21:26:00 495256467
>>495254
>Ты это понял или подсмотрел?
Вики жи
"Для получения характеристик, сравнимых с характеристиками ДВС, приходится применять высокие давления (свыше 100 атм) и особые виды рабочего тела — водород, гелий"
Но мои сомнения вызваны тем, что на ютубе я не видел ни одного рабочего образца реальных размеров - или игрушки как на картинке, или какой то еле двигающийся трактор. Даже с паровыми двигателями есть демонстрации, тут нихуя.
Да, вроде на подводные лодки их ставят, и они норм. Но почему не ставят на наземный транспорт?
>и твой вопрос в таком случае бред
Название треда
Аноним 24/01/20 Птн 21:30:18 495257468
>>495256
Очевидно почёму всегда что-то не используют хоть оно и заебись, ОЧЕНЬ ДОРОГО, экономически неэффективно.
Ты такое не купишь, никто не купит, может 1-2 человека на всю планету, и зачем ради них делать?

Плюс может быть проблема в масштабировании.
Может для движка весом в 5 тонн(какие там на лодки ставят?) Достаточно 100 атмосфер, а для 50 кг, как некоторые современные движки машин, нужно 10,000 атмосфер?

Ну хотя это всё "может-может" пусть кто знает тот ответит, сорри

Но про экономическую эффективность точно.
Аноним 24/01/20 Птн 22:13:57 495261469
145405444311794[...].png (1034Кб, 900x600)
900x600
>>495236
Ты даже высер про чайник Рассела не способен понять, хотя он и у самого Бертрана изначально про бога нихт был, а ты не понял и после двух объяснений от анона выше. Поди ты сам об чайник в говно и макался весь вечер, не зря посты потерли.
Это на плечи утверждающего о существовании чего либо ложится бремя доказательства. Никто не должен опровергать существование души, как и опровергать чайник, тупень, тащишь души - будь готов тащить и доказательство существования оных. Собрался летать и искать чайник в космосе - иди заодно бога с душами поищи, верун. Заебал тупить троллингом.
Аноним 24/01/20 Птн 22:46:35 495262470
>>495261
Ебать ты тупорылый, я хуею.
Аноним 24/01/20 Птн 23:04:10 495263471
>>495234
>как вообще в принципе потенциально может образоваться именно чёрная дыра, т.е. как материя может провалиться за горизонт событий, если для стороннего наблюдателя, т.е. для нас, время процессов возле горизонта бесконечно замедляется.
Ублюдок, мать твою, говно собачье, а ну иди сюда, я тебя сам трахну... Это как лирическое вступление. Для стороннего наблюдателя что именно замедляется, не ешь, подумой! Что вообще сторонний наблюдатель наблюдает? Каким образом? Если не понял - он наблюдает некие сигналы, пусть будем наблюдать фотоны. Это с его точки зрения фотоны все реже приходят, все более в красное уходящие. Для материи падающей в дыру картина другая, ничего там не останавливается.
>Т.е. теоретически, согласно эйнштейновской модели (не уверен, по логике), которая полностью и везде сейчас подтверждается, образование чд невозможно.
Отнюдь. Вещество по собственным часам быстренько упадет, образуется горизонт событий, потом новое вещество, которое не улетит в джеты, тоже спокойно упадет за горизонт, причем довольно быстро, со значительной долей скорости света на спидометре.
>сингулярность внутри вращающейся чёрной дыры представляет собой тороид
Только вроде там кольцо. Ну так она вращается, по математике выходит, что там точка разматывается в кольцо - внутри которого отрицательное расстояние. Там еще и два горизонта событий получаются. Подробно сам читай, там решения Керра, матан, все дела.
Аноним 24/01/20 Птн 23:08:48 495264472
>>495262
Еще жиже перднуть сможешь? Сказать-то нечего, понимаю. Иди тащи интересные тупые вопросы, а то засыпаю нахуй, а мне еще поработать надо, я пока чаю сделаю.
Аноним 24/01/20 Птн 23:32:01 495272473
>>495263
Бле... Ну опять эта хуета...
Ну зачем ты мне про точку зрения материи падающую в чёрную дыру если вопрос чётко задан "для стороннего наблюдателя"?
И естественно вопрос теоретический, допускающий что мы видим всё, вплоть ДО ИСТИННОГО перехода падающей в чд матерри за горизонт.
Такое происходит хотя бы чисто в теории, или согласно современным теорий, образуются(и соответственно существуют) только объекты, бесконечно похожие на чд для НЕ ИДЕАЛЬНОГО стороннего наблюдателя?
Аноним 24/01/20 Птн 23:33:04 495275474
>>495264
Сам доказывай чайник рассела, а пока не доказал-его нет. Тебе-то показать нечего, а я его отсутствие показать могу легко-так что ты сосёшь.
Опущен логически.
Аноним 25/01/20 Суб 00:45:53 495291475
154504841011985[...].png (148Кб, 1024x800)
1024x800
>>495275
Хуя ты тупой, прям аж до мурашек.
>>> существование души доказать никто не смог.
>>а полностью на 100% опровергнуть?
>Как только опровергнешь чайник Рассела - поговорим.
Ты, одноклеточное, и сейчас походу не поймешь, да? Так вот, фитопланктон, тот анон я прямо ткнул в чайник Рассела этого веруна, именно с целью указать ему, кто должен доказывать (а не опровергать). Хватит мои же посылы мне гордо приносить и говорить "я сделалъ", ебанутый чесслово.
>а я его отсутствие показать могу легко
Давай, неси доказательство его отсутствия, лол. Верун ебаный. :3
>>495272
Хуя ты тоже необучаемый, походу это один анон, да? Какая разница, увидим мы фотоны от последних падающих за горизонт остатков звезды или нет, если там уже дыра давно? Горизонт сформировался, за конечное время, мы можем посчитать где он (зная массу), зафиксировали гравитационные волны, а приборы показали, что звезда стала меньше, тусклой, еще тусклее, краснее, потом чернее. Все, фотоны с горизонта краснющие и редкие. Это потому как каждому из них надо "преодолеть" намного больше "расстояния" (искривления) чтобы вылететь, отсюда и рассинхрон часов идет. Ты просто тупишь и так и не понимаешь откуда в "теории относительности" слово "относительность" взялось.
> вплоть ДО ИСТИННОГО перехода падающей в чд матерри за горизонт.
ИСТИННОЕ время и переходы положи дедушке Ньютону, если взялся за ОТО/СТО указывай относительно кого, не забывай, что раз нет дальнодействия, то и нет никакого истинного перехода, ты можешь наблюдать только сигналы, все. Ну и можешь посчитать а) разницу, которые покажут часы для неискривленного пространства/неускоренного объекта относительно искривленного/ускоренного (двух искривленных/ускоренных относительно друг друга), б) как будет выглядеть для наблюдателя мир/объект/траектория, опять же зависящая от выбора системы отсчета.
Аноним 25/01/20 Суб 04:20:47 495313476
>>495291
Ебать ты порвался и в манямирок ушёл, верунский даун, лол
Аноним 25/01/20 Суб 04:22:20 495314477
>>495291
Ой блять заткнись нахуй, что ты там пояснять собрался, ты же тупорылый что пиздец, пиздуй в колхоз говно разбрасывать и в церквушке молится. У тебя один пустой верунский пиздёж.
С наукой в твоих постах ничего не связано.
Аноним 25/01/20 Суб 04:22:44 495315478
>>495291
Фантазёр уёбищный.
Аноним 25/01/20 Суб 08:29:44 495335479
hqdefault (4).jpg (6Кб, 480x360)
480x360
Аноним 25/01/20 Суб 10:33:29 495339480
>>495234
>так это как вообще в принципе потенциально может образоваться именно чёрная дыра, т.е. как материя может провалиться за горизонт событий, если для стороннего наблюдателя, т.е. для нас, время процессов возле горизонта бесконечно замедляется
любой доступный учебник - мизнер\торн\уилер, например, или шутц
вот тоже
https://astronomy.stackexchange.com/questions/2524/would-time-go-by-infinitely-fast-when-crossing-the-event-horizon-of-a-black-hole

ещё https://arxiv.org/abs/1312.6698 может осилишь

>суперновых
сверхновых\supernova
эвристика
https://arxiv.org/pdf/1912.13517.pdf
https://arxiv.org/pdf/1912.05398.pdf
теор модели
https://www.annualreviews.org/doi/abs/10.1146/annurev.astro.38.1.191
http://adsabs.harvard.edu/full/1986ARA%26A..24..205W
https://arxiv.org/pdf/1809.05106.pdf

>щё почему теоретически считается что сингулярность внутри вращающейся чёрной дыры представляет собой тороид, а не точку. Подробно
Visser, M: Lorentzian Wormholes: from Einstein to Hawking
https://journals.aps.org/prl/abstract/10.1103/PhysRevLett.68.2117
https://iopscience.iop.org/article/10.1088/0264-9381/8/5/011/meta

>Ещё интересный вопрос про чд и чёрные дыры задали тут когда тоже писал про падение в чд и невозможность этого, из-за за земедления времени для стороннего наблюдателя.
любой нормальный учебник вроде вайнберга
у виссера тоже есть про это, выше
кщё
https://physics.stackexchange.com/questions/21319/how-can-anything-ever-fall-into-a-black-hole-as-seen-from-an-outside-observer

>>495219
>Ты знаешь, что такое магнитное поле, откуда оно берется и как работает?
> что такое магнитное поле
тебе уже давали ответ на это, тебе он не понравился, это твоё дело. Не знаешь СТО и дифгема - страдай, пенять на зеркало не нужно.
>откуда оно берется
Опять же, тебе дали на это ответ, тебе он не понравился. Не знаешь КТП и понятий калибровочной инвариантности - ешь свой научпоп, ну никто же не осуждает, правда.
> и как работает?
Это было известно ещё более ста лет назад.
Аноним 25/01/20 Суб 10:33:35 495340481
>>495335
Зачем ты фотку своего ануса скинул, вернуский даун?
Аноним 25/01/20 Суб 15:13:16 495369482
К расчитать силу (момент силы) условного самодельного электродвигателя при 100% КПД? Вот намотал я катушку, могу некоторые характеристики посчитать и, даже, индуктивность измерить.
Но что делать с постоянным магнитом? На какие характеристики смотреть и где узнать приблизительные у обычных магнитов? И как расчитать силу их взаимодействия, в конце концов?
Аноним 25/01/20 Суб 19:48:51 495394483
>и как работает?
>Это было известно ещё более ста лет назад.
Всё хорошо кроме этого.

Хотя я не знаю про что он спрашивал, лол.

Да и ответ в принципе подходит но... "Как это работает" двояко понимать можно.
Аноним 25/01/20 Суб 19:50:21 495395484
>>495369
Советую попробовать поискать МЕТОДИЧКИ, на лабараторную работу это не тянет, а вот методички к курсовым-должно быть самое то.
Аноним 25/01/20 Суб 19:53:58 495398485
>>495339
Спасибо, но на английском, не, я не осилю, только на русском.
Аноним 26/01/20 Вск 12:29:59 495466486
Untitled.png (5Кб, 328x291)
328x291
В чем ошибка?
Аноним 26/01/20 Вск 12:38:47 495468487
>>495466
С чего там ошибка? Обычные писульки шазофреника.
Аноним 26/01/20 Вск 12:39:57 495469488
image.png (289Кб, 500x333)
500x333
Аноним 26/01/20 Вск 13:40:34 495472489
>>492883 (OP)
аноны а дайте чтонибудь почитать посмотреть как новые вирусы вообще образуются? ну вы поняли в связи с чем
Аноним 26/01/20 Вск 14:46:21 495475490
>>495472
Так же как и любые новые виды, только быстрее, случайные мутации, особенно у рнк-вирусов (на них не действуют механизмы исправления при репликации днк), перетасовывают гены со схожими вирусами, когда заражают одну клетку, ну и дрейф генов, например когда из-за гибели части популяции какая-то аллель становится более редкой/частой или вообще исчезает/закрепляется.
Аноним 26/01/20 Вск 14:59:39 495476491
3543612.jpg (120Кб, 800x732)
800x732
>>495466
>В чем ошибка?
Твоим формулам не хватает любви.
Аноним 27/01/20 Пнд 12:55:03 495602492
Нужна мне кювета для воды, 90х90х15 см. Подходящую пластиковую тару я не нашел в продаже, значит сделаю сам. Из досок проще всего, но чтобы нормально закрепить на мебельные болты толщину бы 2см, в итоге получившийся короб весить будет много. Предложили сделать из пеноплекса и склеить каким-то супир пупир клеем, дескать максимально легкая конструкция будет.
Заливать я буду на 10см: 90х90х10см=81см3=810л;
Площадь стенок 3600см2. Делим 810л на 3,6 м2, получаем 150мл или 150грамм на 1см2, от чего я усомнился в прочности такой конструкции, на что мне указали на безграмотность.

Это я настолько тупой что не могу посчитать такую хуйню или меня троллят тупостью?
Аноним 27/01/20 Пнд 13:13:15 495605493
>>495602
Ты пеноплекс хоть раз видел? Он пука развалится. Бери орг стекло.
Аноним 27/01/20 Пнд 13:13:38 495606494
Аноним 27/01/20 Пнд 14:15:32 495609495
>>495602
Квадратная конструкция будет непрочной из-за того что на неё давление будет неравномерно действовать(условно говоря).
Делай кругулю бочку.
Круглая выдержит 810 литров даже будучи сделанная из тонкого пластика как бутылки.
Аноним 27/01/20 Пнд 19:20:22 495643496
>>495602
Блядь, вот это я обосрался. В кубическом метре не 100 кубических см, а миллион нахуй, тогда получается не 810л, а 81л и давление уже будет 15 грамм на см2, что вроде норм для пенопласта. Хотя как сказали выше соберу лучше из оргстекла.
Аноним 27/01/20 Пнд 19:38:37 495644497
>>495643
Только круглую делай, иначе разлетится, ты же хуёво склешь.
Аноним 27/01/20 Пнд 19:40:23 495645498
>>495644
Из листового материала намного проще прямоугольное сделать. Кювета без крышки и сверхдавлений там быть не может.
Аноним 27/01/20 Пнд 20:02:13 495647499
Аноним 27/01/20 Пнд 21:17:51 495650500
>>492883 (OP)
Анон, а есть мировая карта распространения следов и территориального нахождения радиоактивных осадков от катастроф и испытаний? Маяки там, Фукусимы, полигоны всякие. Поискал, но везде устаревшая информация.
Аноним 27/01/20 Пнд 21:22:20 495651501
>>495647
Если кювета будет на столе, то только на стенки будет нагрузка и швы. Если подвешана, то я бы лучше сопромат повспоминал прежде чем собирать ибо может днище прогнуться и Ярик, пизда бачок потiк.
Аноним 27/01/20 Пнд 22:53:56 495654502
15797304837860.png (292Кб, 678x391)
678x391
Условно называю "синим" и "красным".
Почему, если закрыть синий вентиль, то батарея не холодеет? Почему красный краник закручивается влево до предела, а вправо он словно бы совсем должен открутиться (но я боюсь крутить больше, чем на 20 оборотов)?
Как перекрыть такую батарею правильно?
Аноним 27/01/20 Пнд 22:55:44 495655503
>>495654
Мне стыдно задавать этот вопрос бате, а все мои знакомые это зумерки-белоручки. Кстати, на дваче нет доски о сантехнике.
Аноним 27/01/20 Пнд 22:58:39 495656504
>>495654
Как долго не холодеет? У воды теплоёмкость большая и надо подождать пока остынет.
Аноним 27/01/20 Пнд 23:29:33 495658505
>>495656
40 минут и никакой разницы. Батарея вроде как чугунная, в гараже. Она слишком горячая и мне плохо в помещении. Подо мной еще люди, опасаюсь нанести им ущерб, а потому вернул в прежнее положение (сейчас морозы, мало ли). Не знаю, опасно ли перекрывать только кран, который перекрывается. Батарея не лопнет/потечет, если перекрыть в неправильном порядке?
Аноним 28/01/20 Втр 00:15:19 495659506
На ночь глядя задумался. Вот есть волна де Бройля, длина равна писечка/импульс. Как пояснял препод - человек тоже волна де Бройля, но у нас импульс огромен (масса велика), константа Планка (она же писечка) мала, писечка делить на большое = вообще маленькая писечка. Так-то мы бы могли как волна интерферировать и дифракциировать, только вот хуй найдёшь щель величиной с такую писечку. Но в чём вопрос. Вот я беру и сажусь. Скорость = 0, импульс = 0. Теперь я волна с бесконечной длиной и дифракциирую на всю ёбаную вселенную. Так, получается?
Аноним 28/01/20 Втр 01:18:33 495660507
>>495659
>Скорость = 0, импульс = 0
В какой системе координат?
Аноним 28/01/20 Втр 03:27:33 495667508
>>495660
Вот этот явно шарит.
Аноним 28/01/20 Втр 05:20:44 495670509
>>495659
>Скорость = 0, импульс = 0
Такого не может быть. Ты точно узнать не можешь скорость и импульс одновременно.
Аноним 28/01/20 Втр 08:43:30 495673510
>>495660
Так, если скорость и импульс зависят от системы координат, то я могу быть одновременно волной разной длины от нуля до бесконечности в зависимости от куда считать? Ебать.
>>495670
В смысле? Скорость умножаю на массу и получаю импульс.
Аноним 28/01/20 Втр 08:57:51 495674511
>>495673
>В смысле? Скорость умножаю на массу и получаю импульс.
У тебя всегда есть погрешность у скорости и у импульса (не путать с погрешностью приборов. В квантовой механике это особая фича, связанная с тем, что если плотность вероятности координаты это дельта функция (точное значение), то по импульсу там полный пиздец (по всей оси размазано), и наоборот аналогично.)
Аноним 28/01/20 Втр 08:59:45 495675512
>>495674
Ну ок. Забудь про скорость, я её написал для импульса, сама по себе она вообще не нужна. Импульс ноль, погрешность координаты хоть по всей вселенной мажь.
Аноним 28/01/20 Втр 09:01:30 495676513
>>495675
>Импульс ноль, погрешность координаты хоть по всей вселенной мажь.
Именно так. Только вот ты не определяешь сам для себя с какой точностью ты "узнаешь" свой импульс.
Аноним 28/01/20 Втр 11:15:27 495680514
Сразу говорю, я тупой.
Жир - это способ запаса энергии? Т.е. мы кушаем и откладываем жирку "на всякий случай"?
Почему организм не контролирует это? Почему мы можем толстеть и толстеть и толстеть? Почему когда мы доходим до определенной точки ожирения организм не перестает откладывать жир? Это баг или задумка?
Аноним 28/01/20 Втр 11:19:05 495681515
>>495680
>Жир - это способ запаса энергии
Да
>>495680
>Это баг или задумка
Ну как те сказать. Вообще задумка - пока есть жрат, надо жрат и откладывать на тот случай, когда не будет жрат. Но щас на нас вывалилось очень много вкусного жрат, к чему "природные" механизмы были не готовы.
Аноним 28/01/20 Втр 19:51:37 495717516
IMG202001281949[...].jpg (51Кб, 818x750)
818x750
IMG202001281949[...].jpg (98Кб, 960x684)
960x684
IMG202001281949[...].jpg (137Кб, 1280x802)
1280x802
IMG202001281950[...].jpg (64Кб, 1024x713)
1024x713
>>495650
Не точное, но схожее.
Аноним 30/01/20 Чтв 14:09:04 496171517
Кто что думает о Боге?
Аноним 22/02/20 Суб 22:21:17 499657518
30 секунда.mp4 (18632Кб, 640x360, 00:04:38)
640x360
>>492883 (OP)
Что изображено на 30 секунде видеорилейтед?

Помню только, что была шебм с этим на доске, и что какой-то анон сравнивал происходящее с живой машиной Тьюринга.
Аноним 23/02/20 Вск 00:36:49 499665519
>>499657
Копирование днк или рнк мутится из днк. РНК-полимераза гугли. Есть ещё ферметны которые шагают и чинят днк. Если найдёшь что путное почитать по теме то кинь в тред.
Аноним 27/02/20 Чтв 01:05:39 500236520
>>499657
Да это хуйня ещё, держи полную версию:
https://www.youtube.com/watch?v=7Hk9jct2ozY
Зырь какая лепота

По моему чувакам, которые охуенно внятно визуализируют то, о чём слышишь только в теории нужно какую-нибудь премию вручать.
Пусть даже символическую, но как-то отмечать такое обязательно необходимо.
Аноним 27/02/20 Чтв 08:27:52 500259521
bambuk.jpg (142Кб, 680x705)
680x705
У меня дома, в вазе с водой, растёт бамбук.
КАК?!
Как это вообще возможно? Почему появляются ростки? Откуда он берёт массу для них? Неужели из одной только воды можно получить всё необходимое для того чтобы появлялись отростки?
Я точно не знаю, но всё-таки мне кажется, что только водорода и кислорода из воды недостаточно, чтобы слепить такую сложную и многосоставную поебеньку как росток.
Аноним 27/02/20 Чтв 14:29:55 500288522
>>500259
А внезапно большую часть массы растения берут не из почвы и не из воды. А из воздуха, точнее из co2. Есть опыт со взвешиванием кадки с землёй с ростком и с выросшим деревцем.
Аноним 10/03/20 Втр 12:38:19 501127523
Существую ли кванты времени?
Аноним 14/03/20 Суб 15:48:44 501491524
Аноним 02/05/20 Суб 20:42:11 505996525
Если я окажусь непосредственно в эпицентре взрыва ядерной бомбы, вот прямо таки облокотившись спиной на боеголовку, то что со мной произойдёт? Меня только испарит в плазму или даже ядра атомов из которых состоит моё тело тоже разъебёт?
Аноним 12/06/20 Птн 23:28:37 509312526
Почему люди облысели? Разве шерсть не идеальна как для холода так и для жары?
Настройки X
Ответить в тред X
15000 [S]
Макс объем: 40Mб, макс кол-во файлов: 4
Кликни/брось файл/ctrl-v
Стикеры X
Избранное / Топ тредов